CIC Practice Test 2

Lakukan tugas rumah & ujian kamu dengan baik sekarang menggunakan Quizwiz!

100. One of the units in an LTAC facility is experiencing an outbreak of crusted scabies. In this situation, when is HCP prophylaxis indicated? a. Prolonged skin-to-skin contact with suspected and confirmed case b. Evidence of infestation such as pruritic cutaneous rash c. All staff, volunteers, and visitors who may have been exposed to a patient with crusted scabies d. Treatment is indicated for all staff on affected units

100. B Evidence of infestation such as pruritic cutaneous rash Rationale: Control measures for an outbreak involving one or more cases of crusted scabies should involve rapid and aggressive detection, diagnosis, infection control, and treatment measures because this form of scabies is so highly transmissible. Unrecognized crusted scabies often is the source of institutional outbreaks of scabies. All staff, volunteers, and visitors who may have been exposed to a patient with crusted scabies, or to clothing, bedding, or furniture used by the patient, should be identified and treated.

101. A biological indicator from one of the sterilizers in Sterile Processing turns positive. The first action should be: a. Retrieve unused items from the load b. Conduct surveillance of involved patients c. Evaluate the sterilizer d. Evaluate staff education of using the sterilizers

101. A Retrieve unused items from the load Rationale: Recalling unused items would prevent further possibility of infection in patients and, therefore, should be done first. A written policy and procedure is needed to address who, when, and how to recall reprocessed items. Evidence of sterilization failures (e.g., positive biological indicators) is the most common reason for a recall. In addition, the policy may include compliance components of the Safe Medical Device Act if failure is noted in the reuse of reprocessed items. At a minimum, a log of items within the sterilized load should be reviewed, unused items in the load should be retrieved for reprocessing, functional evaluation of the involved sterilizer should be completed, and surveillance of involved patients should be initiated

102. An advantage of a case-control study over a cohort study is that a case-control study: a. Is considered less biased than a cohort study b. Provides stronger evidence for a causal association c. Is less time consuming and less expensive d. Is more valid

102. C Is less time consuming and less expensive Rationale: Case-control studies begin with the identification of persons who have the outcome of interest. Then a control group of individuals without the outcome is selected for comparison. Case-control studies are quicker, easier, and cheaper than cohort studies, especially if outcome is rare or has long latency period.

103. Which of the following statements is accurate about adult learners? a. They do not readily speak out if their learning needs are not being met b. They have a preference for academic knowledge rather than practical knowledge c. Most older adults have the same technological capabilities as younger adults d. They are more likely to transfer knowledge to practice if education sessions are interactive

103. D They are more likely to transfer knowledge to practice if education sessions are interactive Rationale: Research shows that adult learners retain and use more of the knowledge they gain if they are encouraged to apply what they learned. Learning is facilitated when:• There is immediate application for the learning.• They participate actively in the learning process.• They can practice new skills or test new knowledge before leaving a learning session When participants are involved in their learning, rather then being passive observers, they are more likely to master the information or concepts presented, apply them to their practice, and retain the information presented.

106. A patient has been admitted to an acute care facility with a diagnosis of rule-out pulmonary TB. Below is the lab report for the sputum testing on this patient. Date and time of sputum collection 03/04 11:43 a.m.Negative for AFB 03/04 8:26 p.m.Negative for AFB 03/05 11:13 a.m.Negative for AFB Can this patient be removed from Airborne Precautions for rule-out pulmonary TB? a. Yes, the patient can be removed from Airborne Precautions, as TB can be ruled out b. No, the patient cannot be removed from Airborne Precautions, because he is infectious c. Yes, the patient can be removed from Airborne Precautions, as he has latent TB and is not infectious d. No, the patient cannot be removed from Airborne Precautions, because the sample collection was not sufficient to rule out pulmonary TB

106. D No, the patient cannot be removed from Airborne Precautions, because the sample collection was not sufficient o rule out pulmonary TB Rationale: Sputum samples for AFB testing for active pulmonary TB must be collected between 8 and 24 hours apart and at least one sample must be an early morning sample. There must be three negative AFB stains to rule out communicable pulmonary TB. In this example the three sample collections are spaced appropriately but there was no early morning sample collected. Therefore, the patient cannot yet be removed from Airborne Precautions.

107. Unused alcohol swabs that are placed on top of a dialysis machine in an active dialysis unit: a. Should be discarded b. May be returned to the common area c. Must be cleaned before being used on another patient d. Should only be used on patients with that dialysis machine

107. A Should be discarded Rationale: There has been transmission of Hepatitis B and Hepatitis C in dialysis settings, which necessitates rigorous infection control practices. Any medications or patient care supplies such as tape, alcohol swabs, and syringes that come into contact with a dialysis machine should be discarded. Other items should be cleaned if possible before being returned to a common area or used on another patient. If cleaning is not possible, then those items should also be discarded.

108. Which interventions are designed to limit the spread of respiratory illnesses in outpatient settings? a. Sterilization monitors b. Aseptic technique c. Respiratory hygiene/cough etiquette d. Symptom-based evaluation

108. C Respiratory hygiene/cough etiquette

109. In preparing for the first meeting of the year for the Infection Prevention and Control Committee, the IP develops an annual report to include HAI trends. Of the choices below, this report should also address: a. A summary of injuries and risk factors for injuries in personnel during the previous year b. Staffing needs of the department o optimize efficiency c. An evaluation of the surveillance program providing an assessment of its usefulness to the healthcare facility in preventing and controlling infections d. A line list of all HAIs for the previous year and risk- reduction strategies

109. C An evaluation of the surveillance program providing an assessment of its usefulness to the healthcare facility in preventing and controlling infections Rationale: An annual evaluation of the infection prevention program is important to outline achievements and activities of the program and describe support requirements. The value of the infection prevention program to the organization should be emphasized, along with patient outcomes and cost savings.

110. The IP has identified a cluster of Candida bloodstream infections in two adjoining ICUs. She wants to look at risk factors that these patients may have had in common. Which study design would she use? a. Cross-sectional study b. Cohort study c. Case-control study d. Clinical trial

110. C Case-control study Rationale: A case-control study is designed to help determine if an exposure is associated with an outcome (i.e., disease or condition of interest). Case-control studies begin with the identification of persons who have the outcome of interest. Then a control group of individuals without the outcome is selected for comparison. For example, in a study to determine risk factors for healthcare-associated bacteremia, patients with bacteremia are identifiedand compared with a control group of hospitalized patients without bacteremia; medical records are reviewed to determine exposures to various factors, such as IV devices, invasive monitoring devices, prior infections, and immunocompetence.

111. An IP is planning an educational program for hospital nursing staff to improve compliance with infection prevention strategies. Please choose the approach that should be used to ensure an effective learning experience for those who attend. a. Provide food and drinks to all who attend as well as sufficient beaks and have a raffle o entice attendance b. Plan the educational offering by developing goals and objectives and determining the teaching method to use c. Use a slide presentation that periodically provides humor to the presentation to ensure attention d. Provide continuing educational credits and handouts for future reference

111. B Plan the educational offering by developing goals and objectives and determining the teaching method to use Rationale: The educator controls the learning experience with a well-defined plan using goals, objectives, and appropriate teaching methods. Goals are statements that communicate the intent of the curriculum and provide a direction for planning the education session. Expectations are clearly defined in terms of time and available resources.

112. Which of the four clinical variables that determine the probability of infection would the IP be targeting when reviewing preoperative chlorhexidine gluconate (CHG) bathing instructions that have been given to the preoperative patient population? a. Inoculum of bacteria b. Virulence of bacteria c. Adjuvants in the microenvironment d. Efficiency of host defenses

112. A Inoculum of bacteria Rationale: The probability of infection is determined by the interaction of four clinical variables: (1) inoculum of bacteria, (2) virulence of bacteria, (3) adjuvants in the microenvironment, and (4) efficiency of host defenses. The risk for SSI is related to the number of microorganisms contaminating the wound. A preoperative antiseptic shower or bath decreases skin microbial colony counts. Clinical studies have documented that multiple applications of 2 or 4 percent CHG using a standardized protocol results in high skin surface concentrations sufficient o inhibit/kill skin colonizing flora, including MRSA.

113. Which of the following processes should take place after an adverse event has occurred? a. Failure mode effect analysis (FMEA) b. Root cause analysis (RCA) c. Strengths-weaknesses-opportunities-threats analysis (SWOT) d. Infection control risk assessment (ICRA)

113. B Root cause analysis (RCA) Rationale: The RCA process takes a retrospective look at adverse outcomes and determines what happened, why it happened, and what an organization can do to prevent the situation from recurring. A thorough RCA determines: (1) human and other factors; (2) the process or system involved; (3) underlying causes and effects of the process; and (4) the risks and potential contributions to failure or adverse results.

114. What is the annual sharps injury rate per 100 full-time equivalents (FTEs) in a facility that has 18 sharp injuries and 800 full-time employees?a. 0.02 b. 0.23 c. 2.25 d. 22.5

114. C 2.25 Rationale: A rate measures the probability of occurrence (i.e., frequency) in a population of some particular event, such as cases of disease or deaths. A rate provides a means of comparing the occurrence of an event in one population to similar populations by adjusting for differences in population sizes.

115. An adult patient is admitted through the Emergency Department with a 1-day history of rash, fever, and cough. The attending physician is concerned that the patient has measles, as the patient cannot recall whether he has had measles disease or vaccine in the past. The serum sample does not contain measles-specific IgG or IgM antibodies. What is the significance of this finding a. The patient does not have measles infection b. The patient has had the measles vaccine in the past c. The patient has a history of measles disease d. The patient might be infected with measles

115. D The patient might be infected with measles Rationale: If the patient had been vaccinated or had a history of measles disease, his serum sample would most likely have contained measles-specific IgG antibody. We cannot assume that the patient does not have measles infection at only 24 hours after the onset of rash, as there might not have been enough time yet for the immune response to develop. It can take up to 72 hours after the onset of rash for IgM to be produced in response to measles infection, so this patient might be infected.

116. Primary components of an initial outbreak investigation are: 1) Preparing a line list and epidemic curve 2) Notifying key partners about the investigation 3) Continuing case finding 4) Initiating an analytic study a. 1, 4 b. 2, 3 c. 1, 2 d. 3, 4

116. C 1, 2 Rationale: In general, outbreak investigations can be divided into two major sections, the initial investigation and the follow-up investigation, each with multiple components. The primary components of the initial investigation include the following: • Confirming the presence of an outbreak • Alerting key partners about the investigation • Performing a literature review • Establishing a preliminary case definition • Developing a methodology for case finding • Preparing an initial line list and epidemic curve • Observing and reviewing potentially implicated patient care activities • Considering whether environmental sampling should be preformed • Implementing initial control measures

117. An IP has been asked to present an educational program to a group of HCP who are culturally different from him. What is the best approach to developing an effective program in this case? a. Develop the program without taking cultural diversity into account, but allow the learners to ask questions while he is teaching b. Be aware of potential cultural differences, but do not incorporate them into the program c. Outline the major points of the educational program, and ask the audience to work together to identify culturally appropriate examples of the major points d. There is no need to take cultural diversity into account because that does not change the learning objectives of the educational program

117. C Outline the major points of the educational program, and ask the audience to work together to identify culturally appropriate examples of the major points Rationale: An increasingly diverse workforce has driven the need for increased attention to transcultural competence in training programs. Cultural diversity and a mixed workforce are regarded as organizational strengths. People who do not think alike help to create a competitive advantage when problem solving. Concepts of transcultural care need to be incorporated into successful educational activities. Cultural backgrounds will affect the ability of the learner to participate in learning activities and accommodate new skills and ideas. Transcultural education will encompass different perceptions based on geography, gender, religion, social status, age, sexual orientation, and ethnic diversity. Answer C is correct, as this will allow participants to apply the educational content in a manner that is culturally familiar to them.

118. The IP is helping develop a large-scale project to address SSIs. Which of the following tools will the IP utilize that helps secure expert judgment prior to launching the project? a. Delphi technique b. Peer review c. Fisher exact test d. Meta-analysis

118. A Delphi technique Rationale: The Delphi technique is a structured communication method that solicits opinions from a panel of experts who answer questionnaires in two or more rounds. After each round, the responses are summarized and redistributed for discussion. The experts are encouraged to revise their earlier answers in light of the replies of other members of their panel. Common trends are identified, outliers ae examined and a consensus is reached. Delphi is based on the principle that forecasts (or decisions) from a structured group of individuals are more accurate than those from unstructured groups.

119. The use of influenza vaccines in school-aged children to decrease the number of cases in the community uses the principle of: a. Innate immunity b. Passive immunity c. Herd immunity d. Epidemic immunity

119. C Herd immunity Rationale: Herd or community immunity describes a situation in which a sufficient proportion of a population is immune to an infectious disease (through vaccination and/or prior illness) to make its spread from person to person unlikely. Even individuals not vaccinated are offered some protection because the disease has little opportunity to spread within the community.

20. Several HCP have been exposed to a patient with untreated, active pulmonary TB. Which is the best option for follow-up after this exposure? a. TSTs should be administered at the time of exposure; If these are negative, then no further follow-up is needed b. TSTs should be administered at the time of exposure and repeated at 12 weeks post exposure; converters without symptoms should be excluded from work and treated immediately c. TSTs should be administered at the time of exposure and repeated at 12 weeks post exposure; converters with symptoms should follow up with a chest x-ray d. TSTs and chest x-rays should be administered at the time of exposure and repeated at 12 weeks post exposure

120. C TSTs should be administered at the time of exposure and repeated at 12 weeks post exposure; converters with symptoms should follow up with a chest X-ray Rationale: TSTs should be administered at the time of exposure to establish the baseline TST reaction. Follow-up TST should occur at 12 weeks post exposure to determine whether infection has occurred. If HCP are symptomatic in conjunction with a TST conversion, they should be screened for active TB by chest X-ray.

121. Microorganisms are grown on culture media made of an agar base. Additives to media vary according to growth requirements of organisms and/or the desire to select out a specific organism. Fastidious organisms require _________ media and _________ media are used to inhibit normal commensals. 1) Differential 2) Enrichment 3) Selective 4) Nutrient broth 5) Synthetic sheep blood agar a. 1, 3 b. 2, 3 c. 3, 4 d. 5, 1

121. B 2, 3 Rationale: A growth medium or culture medium is a liquid or gel designed to support the growth of microorganisms or cells. There are several categories of growth media, including (1) nutrient agar, a general-purpose growth medium that supports the growth of a wide variety of bacteria (e.g., trypticase soy agar with 5 percent sheep blood); (2) enrichment medium, which contains special nutrients necessary for the growth of hard-to-grow (fastidious) bacteria (e.g., chocolate agar for the growth of Neisseria meningitidis); (3) selective media that contain chemicals or antibiotics designed to inhibit normal commensals, allowing organisms of interest to grow (e.g., bismuth sulfate agar for the isolation of Salmonella spp.); and (4) differential media that promote the differentiation of specific oganisms while inhibiting others (e.g., acetate agar to differentiate E. coli from Shigella).

122. A patient is concerned that there might be microbes in the facility that are resistant to the environmental disinfection products in use. How do you respond to this concern? a. Inform her that while microbes can develop resistance to antimicrobials, they cannot become resistant to disinfectants b. Assure her that the disinfection products in use are ones that microbes have not developed resistance to yet c. Inform her that while reduced susceptibility of microbes to disinfectants can occur, the antimicrobial level of disinfectants used is still sufficient o inactivate those microbes d. Assure her that your facility uses special environmental disinfection procedures for all antimicrobial resistant organisms

122. C Inform her that while reduced susceptibility of microbes to disinfectants can occur, the level of disinfectant used is still sufficient o inactivate those microbes Rationale: As with resistance to antibiotics, microbes can develop altered sensitivity to environmental disinfectant products though spontaneous mutation in the bacterial genome, transmission of transposable resistance genes to the chromosome from a plasmid and vice versa, and transfer of resistance genes on plasmids between microbes. However, decreased sensitivity or increased tolerance to environmental disinfectant products does not render the microbes resistant to these products and the concentrations of product used in the healthcare facility remain cidal for even less sensitive organisms.

123. An IP has monitored the rate of hand hygiene compliance among different nursing units. She finds that the ICU staff's compliance is less than satisfactory even after providing conveniently located hand hygiene dispensers. She shares this feedback with the staff. Of the choices below, what other information should be included in her discussion with the staff to improve hand hygiene compliance? 1) The number of patients on the unit during the monitoring period compared to the number of staff, as well as the nurse to patient ratio 2) Information about hand contamination 3) Information about the association between hand hygiene practices and the transmission of infection 4) The effects of hand hygiene products on skin a. 1, 2, 3 b. 2, 3, 4 c. 1, 3, 4 d. 1, 2, 4

123. B 2, 3, 4 Rationale: To improve hand hygiene compliance, HCP should be provided with evidence-based information about hand contamination, the effects of hand hygiene products on the physiology of normal skin, and the association between hand hygiene practices and transmission of infection. The process of change is complex, and single interventions often fail; therefore, a multimodal, multidisciplinary strategy is necessary to change and improve hand hygiene practices.

124. The IP is called to the day care center for a possible outbreak of Hepatitis A. The Public Health Nurse is assisting her in investigating the outbreak. Prophylactic administration of immunoglobulin to the day-care workers and non infected children would be an example of: a. Passive immunity b. Active immunity c. Herd immunity d. Nonspecific immunity

124. A Passive immunity Rationale: Hepatitis A, caused by infection with HAV, has an incubation period of approximately 28 days (range: 15 to 50 days). HAV replicates in the liver and is shed in high concentrations in feces from 2 weeks before to 1 week after the onset of clinical illness. HAV infection produces a self-limited disease that does not result in chronic infection or chronic liver disease. Current CDC guidelines recommend that persons who have recently been exposed to HAV and who have not been vaccinated previously be administered a single dose of single-antigen Hepatitis A vaccine or Immunoglobulin (IG) (0.02 mL/kg) as soon as possible, within 2 weeks after exposure. IG is an example of passive immunity, which is provided when a person is given antibodies to a disease rather than producing them through his or her own immune system.

125. Which of the following would be an acceptable use of immediate-use sterilization for a surgical instrument? a. To re sterilize a surgical instrument that was dropped on the floor and or which no replacement is available b. To sterilize an expensive surgical tool so that additional tools do not need to be purchased c. To sterilize instruments that are not heat tolerant d. To sterilize an instrument quickly so that a surgeon does not have to wait for a replacement to be brought to the OR

125. A To re sterilize a surgical instrument that was dropped on the floor and for which no replacement is available Rationale: The Association for the Advancement of Medical Instrumentation (AAMI) defines immediate-use steam sterilization (IUSS) as the "Process designed for cleaning, steam sterilization, and delivery of patient care items for immediate use previously known as flash sterilization." The Association for Operating Room Nurses (AORN) states that IUSS should be kept to a minimum and should only be used in select clinical situations and in a controlled manner. IUSS should only be used when there is insufficient time to process the preferred wrapped or container methods intended for terminal sterilization, and it should not be used as a substitute for sufficient instrument inventory.

126. The IP has implemented a new educational program focusing on teaching safe injection practices to the staff nurses. Several months after the program is initiated, the IP is notified of potential outbreak issue—a nurse used a medication vial for more than one patient. Which of the following quality tools would be most useful to investigate this incident? a. Value stream mapping (VSM) b. RCA c. SWOT d. FMEA

126. B RCA Rationale: The RCA process takes a retrospective look at adverse outcomes and determines what happened, why it happened, and what an organization can do to prevent the situation from recurring. The RCA process avoids individual blame, considers human factors engineering, and analyzes redesign for a safer system. A thorough RCA determines (1) human and other factors; (2) the process or system involved; (3) underlying causes and effects of the process; and (4) the risks and potential contributions to failure or adverse results.

127. A patient is admitted to the hospital with severe community-acquired pneumonia (CAP). His symptoms include hemoptysis and a multilobar infiltrate seen on chest radiograph. The patient is recovering from a recent influenza virus infection. What organism should be suspected? a. Moraxella catarrhalis b. Streptococcus pneumoniae c. MRSA d. Haemophilus influenza

127. C MRSA Rationale: Community-associated MRSA (CA-MRSA) strains are primarily associated with skin and soft tissue infections; however, they are increasingly causing more invasive infections, including severe CAP. CA-MRSA should be suspected in patients with severe CAP plus hemoptysis, multilobar or cavitary infiltate seen on chest radiograph, or neutropenia.

128. An IP notes that there appears to be an increase in positive AFB results for a 2-week time period. She recalls that there had been a confirmed case of Mycobacterium tuberculosis in a college student at the beginning of the time period. In all, six patients have culture results that confirm TB. after conferring with the Microbiology supervisor, a break in the lab's procedure is identified. the buffer solution that is used in these tests was contaminated with the initial patient's TB. This conclusion is confirmed with polymerase chain reaction (PCR) testing of the isolates. What would this type of outbreak be called? a. TB outbreak b. Pseudo-outbreak c. Epidemic d. Propagated outbreak

128. B Pseudo-outbreak Rationale: A pseudo-outbreak is defined as an episode of increased disease incidence due to enhanced surveillance or other factor not related to the disease under study. It is generally applied to situations in which there is a rise in test results (e.g., positive microbiology cultures) without actual clinical disease.

129. Which of the following would be best studied via qualitative research? a. The effect of preoperative bathing with CHG on SSI rates b. Declining influenza vaccination rates in HCP c. MRSA colonization rates in dialysis patients d. Reoccurrence of C. difficil in nursing home patients

129. B Declining influenza accination rates in HCP Rationale: Qualitative research methods can be used to study phenomena of interest to IPs, including HCP adherence to infection prevention recommendations (e.g., immunization schedules, hand hygiene, and safe surgical techniques). By using qualitative methods, IPs can systematically identify variables and relationships among variables that influence the practices and behavior of HCP. Qualitative research can be used to determine why HCP are not receiving the flu vaccine.

130. A facility has decided to engage in animal-assisted activities/animal-assisted therapy, and the IP has been asked to develop infection prevention guidelines related to these activities. Of the choices below, which should be included in the guidelines? a. The types of animals visiting a patient must be limited to small dogs less than 20 lbs. with the proper temperament b. Animals must be screened to ensure that they are healthy and that their immunizations are current c. A patient's personal pet may interact with other patients d. Service animals may enter the operating room

130. B Animals must be screened to ensure that they are healthy and that their immunizations are current

131. The IP lives in a community of 100,000 persons. There have been more than 1,000 cases of Hepatitis C with 200 resultant deaths in 1 year. The case fatality rate for this is: a. 10 percent b. 800 per 10,000 c. 20 percent d. 200 per 100,000

131. C 20 percent Rationale: The case-fatality rate (CFR) is the proportion of persons with a particular condition (cases) who die from that condition. It is a measure of the severity of the condition. The formula is: CFR = x/y × k Where:x = Number of cause-specific deaths among the incident cases y = Number of cause-specific deaths among the incident cases k = Usually an assigned value of 100 when calculating CFR A CFR is conventionally expressed as a percentage and represents a measure of risk. CFRs are most often used for diseases with discrete, limited time courses, such as outbreaks of acute infections. The CFR for this scenario is calculated as (200 ÷ 1000) × 100 = 20 percent

132. Which of the following is not an example of syndromic surveillance? a. Number of patients seen in the Emergency Room with influenza-like illness b. Number of purchases of over-the-county flu remedy medications c. Number of school absences related to illness d. Number of new cancer diagnoses reported to the cancer registry

132. D Number of new cancer diagnoses reported to the cancer registry Rationale: Syndromic surveillance now includes any indicator that might signal an increase in illness in the community. Some examples of data that could be collected and analyzed as part of a syndromic surveillance program include (1) number of patients seen in an emergency department; (2) number of patients presenting to the emergency department with influenza-like illness as their chief symptom; (3) number of patients admitted to a hospital; (4) number of emergency medical services or ambulance runs performed each day, week, month, or other time period; (5) number of purchases of over-the-counter flu remedies; (6) number of purchases of over-the-counter diarrhea medications; or (7) other data available from healthcare facilities or agencies that may indicate a change or trend in the community.

133. Which of the following is used to determine efficacy of antimicrobials on a particular pathogen? a. KOH preparation b. Methylene blue stain c. Rapid Plasma Reagin d. Kirby-Bauer test

133. D Kirby-Bauer test Rationale: Kirby-Bauer antibiotic testing (KB testing or disc diffusion antibiotic sensitivity testing) is a method to determine the sensitivity of microorganisms to specific antimicrobial drugs; greater drug efficacy yields larger microbe-free zones surrounding drug-containing disks after overnight growth on solid media.

134. The IP must prepare a cost-benefit analysis (CBA) for the proposed purchase of a new electronic surveillance system for the hospital. Which statement about the CBA is correct? a. All measures are expressed in monetary (US dollar) terms b. Only costs are expressed in monetary (US dollar) terms c. CBA is never based on existing financial data d. The monetary value of possible benefits is ofen highly speculative

134. A All measures are expressed in monetary (US dollar) terms Rationale: CBA is a systematic method of estimating the strengths and weaknesses of alternatives that satisfy transactions, activities, or functional requirements for a business. It is a technique that is used to determine options that provide the best approach for adoption and practice in terms of benefits such as labo, time, and cost savings. CBAs can help determine which alternative is a sound investment or decision by comparing the total expected cost of each option against the total expected benefits. he comparison shows whether the benefits outeigh the costs and by how much.

135. A dialysis patient who is known to be positive for Hepatitis B has been admitted to the surgical floor or a surgical procedure. The patient will require dialysis three times a week while he is in the hospital. The inpatient dialysis unit does not have an isolation room and is very small. The dialysis nurse has requested the IP's assistance in identifying the best location to dialyze the patient. Which of the following should the IP recommend? a. Transfer the patient to an isolation room in the outpatient unit b. Perform the treatment in the inpatient unit c. Perform the treatment in the patient's hospital room d. Transfer the patient to an isolation room in the ICU

135. C Perform the treatment in the patient's hospital room Rationale: While hospitalized, HBsAg-positive chronic hemodialysis patients should undergo dialysis in a separate room and use separate machines, equipment, instruments, supplies, and medications designed only for HBsAg-positive patients. While HBsAg-positive patients are receiving dialysis, staff members who are caring for them should not care for susceptible patients.

99. Five cases of prosthetic valve endocarditis caused by Staphylococcus epidermidis are observed in one hospital. Of the following available methods, which is best for determining whether all fie isolates were derived from a single source? a. Serotyping b. Pulsed-field gel electrophoresis c. Antimicrobial susceptibility testing d. Bacteriophage typing

99. B Pulsed-field gel electophoresis Rationale: Pulsed-field gel electophoresis (PFGE) begins with the lysis of organisms and digestion of their chromosomal DNA with restriction enzymes. The fragments are separated into a pattern of discrete bands by switching the direction of the electrical current. This pattern serves as a "bar code" of the bacterial chromosome that can be used to assess the relatedness of different clinical isolates. This test method may be used with any organism from which chromosomal DNA can be properly isolated; it has been used with a wide variety of bacterial pathogens to assess epidemiological interrelationships. PFGE is probably the most widely used method for molecular epidemiology and is generally considered to be the gold standard for most clinically important organisms.

no

no

no

revisit

revisit

revisit

65. Which of the following is the primary method to prevent influenza? a. Annual vaccination b. Hand washing c. Droplet Precautions d. Promotion of respiratory hygiene/cough etiquette

A Annual vaccination Rationale: Vaccination is the primary method for preventing influenza and its complications. Recommendations for vaccine and antiviral drug use are published regularly by the ACIP. All persons aged 6 months and older should be vaccinated annually unless specific medical contraindications exist.

The IP has been notified that three patients with possible pneumonic plague have been admitted to the Emergency Department. The IP recommends which of the following strategies?a. A surgical mask worn within 3 feet of patient, door may be open b. Negative pressure isolation room with use of N95 respirators c. Normal ventilation, but door must remain closed and N95 respirator mask worn d. No masks are required, but patient must be placed in private room and contacts should be treated for exposure

A A surgical mask worn within 3 feet of patient, door may be open Rationale: Pneumonic plague is the least common form of naturally occurring disease and also the most severe. The mortality rate is nearly 100 percent in untreated cases and almost 60 percent even when treated. In a bioterrorism event, primary pneumonic plague is most likely to occur because it results from the inhalation of aerosolized bacterial particles. The incubation period for pneumonic plague is 1 to 6 days, but most commonly occurs 2 to 4 days after exposure. Clinical features for pneumonic plague are similar to symptoms for the other forms of plague: nonspecific influenza-le symptoms such as fever, chills, body aches, malaise, headache, and gastrointestinal distress such as nausea, vomiting, diarrhea, and abdominal pain. Patients typically progress from feeling well to having severe pneumonia with cough, chest pain, shortness of breath, and stridor within 24 hours. Pneumonic plague can be spread from person to person. Transmission occurs by respiratory droplets. Patients with pneumonic plague require Droplet Precautions. Special air handling or negative pressure rooms are not indicated. Droplet Precautions (in addition to Standard Precautions) require that patients be placed in private rooms or cohorted, wearing a mask when working within 3 feet of the patient (logistically, some hospitals may want to implement the wearing of a mask to enter the room). Patient transport should be minimized to essential purposes only, and if movement is necessary, the patient should wear a surgical mask to minimize dispersal of droplets. Isolation generally can be discontinued after 48 hours of appropriate antimicrobial therapy. However, isolation should never be discontinued if the patient is not clinically improving. It is possible that the terrorists will genetically alter the strain of plague they release and make it antibiotic resistant. If an antibiotic-resistant strain is used in a bioterrorism attack and isolation is discontinued after 48 hours, the patient would remain infectious and thus constitute a risk to staff, patients, and visitors. Isolation should not be discontinued until 48 hours of appropriate therapy and when the patient is showing signs of clinical improvement.

28. Which of the following is an advantage of orthophthalaldehyde (OPA) over glutaraldehyde? a. OPA does not require activation b. OPA is compatible with many more materials than glutaraldehyde c. It does not stain skin d. No personal protective equipment is needed to handle OPA

28. A OPA does not require activation Rationale: There are several advantages of OPA over glutaraldehyde. OPA requires no activation, is stable over a range of pH values, does not irritate the eyes and nose, and does not have a strong odor. Both OPA and glutaraldehyde are compatible with many types of materials.

42. When using heat to treat a room for bed bugs, the temperature must be: a. 85°F or 29°C for one hour b. 110°F or 43°C for 90 minutes c. 125°F or 51°C for two hours d. 118°F or 48°C for one hour

42. D 118°F or 48°C for one hour Rationale: Bed bugs are challenging to eradicate. Some pest control firms utilize specialized heating equipment to de-infest furnishings, rooms, and entire dwellings. The procedure involves heating up the infested item or area to temperatures lethal to bed bugs. Portable heaters and fans are used to treat the room with high heat (118°F or 48°C for one hour).

105. Which of the following would be a good tool to prepare for a Joint Commission visit? a. Root cause analysis b. Gap analysis c. Multivoting d. Plan, Do, Study, Act

105. B Gap analysis Rationale: Business and quality professionals describe a gap analysis as a technique to determine the steps to take to move from a current state to a desired future state. A gap analysis begins with (1) listing characteristic factors, such as attributes, competencies, or performance levels of the present situation (what is); (2) listing factors required to achieve the future objectives (what should be); and (3) identifying the highlights or "gaps" that exist within the process and that must be filled o meet a goal or achieve standard compliance. Literature also refers to a gap analysis as a need-gap analysis, needs analysis, and needs assessment (see Table PE2-3 for sample).

15. Which of the following statements are true regarding consent to immunization? 1) Some states allow personal belief exemptions 2) Federal regulations require Informed consent 3) All states allow medical exemptions for persons with medical contraindications to vaccination 4) Vaccine recipients must receive Vaccine Information Statements (VISs) a. 1, 2, 3 b. 2, 3, 4 c. 1, 3, 4 d. 1, 2, 4

15. C 1, 3, 4 Rationale: HCP are required by the National Childhood Vaccine Injury Act (NCVIA) to provide a copy of the Vaccine Information Statement to either the adult recipient or to the child's parent/legal representative. All states allow medical exemptions for persons who have medical contraindication to vaccination. Most states allow religious exemptions and some allow philosophical/personal belief exemptions. There is no federal requirement for informed consent relating to immunization.

22. Contact Precautions for a patient with scabies can be discontinued when the patient has received effective treatment for: a. 10 days b. 7 days c. 48 hours d. 24 hours

22. D 24 hours Rationale: For hospitalized patients, Contact Precautions are required for 24 hours after the start of effective treatment. Twenty-four hours may be insufficient in cases of crusted scabies because viable mites can remain on the patient after a single treatment; in this case an alternative isolation approach is suggested in institutional outbreaks: 10-day quarantine of the index patient.

10. While rounding in an ambulatory care center, an IP discovers that healthcare personnel (HCP) have been using single-dose vials (SDVs) of lidocaine for multiple patients. She informs the clinic manager that the practice must end immediately. Which of the following statements about SDVs should the IP include in her explanation to the clinic manager? 1) SDVs lack antimicrobial preservatives 2) Inappropriate use of SDVs can lead to contamination 3) A needleless access device (spike) must be applied when reusing an SDV 4) All medications from an SDV must be prepared in a pharmacy a. 1, 2 b. 2, 3 c. 3, 4 d. 2, 4

0. A 1, 2 Rationale: The CDC's guidelines call for medications labeled as "single dose" or "single use" to be used for only one patient. This practice protects patients from life-threatening infections that occur when medications get contaminated from unsafe use. Vials labeled by the manufacturer as "single dose" or "single use" should only be used for a single patient. These medications typically lack antimicrobial preservatives and can become contaminated and serve as a source of infection when they are used inappropriately.

1. The infection preventionist (IP) on the Antimicrobial Stewardship Team is thinking of ways that he can support efforts and add to the success of the team in decreasing antimicrobial resistance. Some of the activities that he can do to help with the mission of the team include: 1) Calculate multidrug-resistant organism (MDRO) infection rates 2) Detect asymptomatic carriers using active surveillance cultures 3) Use molecular typing for investigating outbreaks 4) Collect environmental cultures of isolation rooms a. 2, 3, 4 b. 1, 3, 4 c. 1, 2, 4 d. 1, 2, 3

1. D 1, 2, 3 Rationale: Surveillance of MDROs is critical to an antimicrobial stewardship program. IPs monitoring microbiology isolates to detect prevalence and emergence of MDROs. IPs may also support antimicrobial stewardship efforts in the following ways:• Calculate MDRO incidence on the basis of clinical culture results• Calculate MDRO infection rates• Use molecular typing for investigating outbreaks• Detect asymptomatic carriers using active surveillance cultures

104. An IP has data on the number of bloodstream infections per central line days in the Cardiac ICU. This is an example of which type of data? 1) Discrete data 2) Categorical data 3) Noncategorical data 4) Continuous data a. 1, 2 b. 1, 3 c. 1, 4 d. 2, 4

104. B 1, 3 Rationale: Discrete data contain whole numbers and are mutually exclusive (e.g., infected or not infected, male or female, blood type). Discrete data can be categorical or noncategorical. Categorical data can count both the number of events/occurrences and the number of nonevents/nonoccurrences (e.g., for 10 SSIs in 100 surgical cases, there are 10 events [SSIs] and 90 nonevents [no SSIs]). Noncategorical data can count the events/occurrences but not the nonevents/nonoccurrences (e.g., number of patient falls per 1,000 patient days). With noncategorical data, the number at risk can be identified, but the actual number of "no infections" or "no falls" among those at risk cannot be identified. continuous data contain information that can be measured on a continuum or scale and can have numeric values between the minimum and maximum value (a continuum) (e.g., age; serum cholesterol level; temperature, such as 98.6°F, 98.7°F, and 98.8°F; infection rates); continuous data require the process of measuring, rather than counting, and may contain whole numbers, decimals, or percentages. The type of data in this scenario is an example of both discrete data because it contains whole numbers and noncategorical data because it is only counting the events (i.e. infections), not the nonevents.

11. A patient is admitted with measles and placed on Airborne Isolation. How many days after symptom onset would the characteristic blotchy red rash appear? a. On days 1-2 b. On days 7-10 c. On days 3-7 d. On days 21-25

11. C On days 3-7 Rationale: Measles symptoms generally appear in two stages. In the first stage, which lasts 2 to 4 days, the individual may have a runny nose, cough, and a slight fever. The eyes may become reddened and sensitive to light, while the fever gradually rises each day, often peaking as high as 103° to 105°F. Koplik spots (small bluish white spots surrounded by a reddish area) may also appear on the gums and inside of the cheeks. The second stage begins on the third to seventh day and consists of a red blotchy rash lasting 5 to 6 days. The rash usually begins on the face and then spreads downward and outward, reaching the hands and feet. The rash fades in the same order that it appeared, from head to extremities.

12. The annual education budget for the Infection Prevention Department is $1,650.00. In October, the Infection Prevention Manager allocated 20 percent of the department education budget towards resources for the annual flu shot program. However, in November, the financial report indicates that only 15 percent was spent. How much of the budgeted amount remains unspent? a. $330.00 b. $247.50 c. $82.50 d. $66.00

12. C $82.50 Rationale: A budget is a quantitative expression of a plan for a defined period of time. It may include planned sales volumes and revenues, resource quantities, costs and expenses, assets, liabilities, and cash folws. It expresses strategic plans of business units, organizations, activities, or events in measurable terms. The manager's budget for the flu shot program is 20 percent of $1,650 (1,650 × 0.20), or $330. However, only 15 percent (1,650 × 0.15), or $247.50 was spent. This leaves a remainder of $82.50.

13. An IP is assisting local public health with a Hepatitis A outbreak in the community. She has been asked to contact the Health Department with any patients who are admitted to her facility with a test positive for Hepatitis A virus (HAV). Patients who test positive in the acute phase of the illness will have a positive: a. Immunoglobulin G (IgG) anti-HAV b. Immunoglobulin M (IgM) anti-HAV c. Immunoglobulin A (IgA) anti-HAV d. IgG, IgM anti-HAV

13. B Immunoglobulin M (IgM) anti-HAV Rationale: HAV is of the genus Hepatovirus in the family Picornaviridae of enteroviruses. It is a nonenveloped, 27-nm single-stranded RNA virus. HAV is transmitted primarily by the fecal-oral route, facilitated by intimate personal contact (household, sexual, etc.), poor hygiene, unsanitary conditions, or contaminated water, milk, or food, especially raw shellfish. Clinical eatures of acute hepatitis are not specific or HAV infection, so serological diagnosis is necessary. Demonstration of the IgM antibodies against HAV (IgM anti-HAV) in the serum of acutely or recently ill patients establishes the diagnosis.

14. A patient from a long-term acute care (LTAC) facility is being transferred to an acute care facility for an outpatient procedure. The patient has a history of cryptococcal meningitis. The outpatient department asks the IP what type of Isolation Precautions the patient requires. The IP informs them that the patient requires: a. Airborne Precautions b. Droplet Precautions c. Standard Precautions d. Contact Precautions

13. B Immunoglobulin M (IgM) anti-HAV Rationale: HAV is of the genus Hepatovirus in the family Picornaviridae of enteroviruses. It is a nonenveloped, 27-nm single-stranded RNA virus. HAV is transmitted primarily by the fecal-oral route, facilitated by intimate personal contact (household, sexual, etc.), poor hygiene, unsanitary conditions, or contaminated water, milk, or food, especially raw shellfish. Clinical features of acute hepatitis are not specific or HAV infection, so serological diagnosis is necessary. Demonstration of the IgM antibodies against HAV (IgM anti-HAV) in the serum of acutely or recently ill patients establishes the diagnosis.

16. An inspection of the Sterile Processing Department reveals several incorrect practices. Which of the following would be a correct practice? a. Ensuring hinged instruments are cleaned with the hinge closed b. Flushing instruments with saline c. Daily use of a biological indicator in the sterilizer d. Transporting contaminated instruments in a permeable container

16. C Daily use of a biological indicator in the sterilizer Rationale: Steam sterilizers should be routinely tested at least weekly—preferably daily—with a biological indicator process challenge device. If a sterilizer is used frequently (e.g., several loads per day), daily use of biological indicators allows earlier discovery of equipment malfunctions or procedural errors and thus minimizes the extent of patient surveillance and product recall needed in the event of a positive biological indicator.

revisit

17. B 2, 3 Rationale: By dividing the total number of patients screened by the total number of patients in each respective category, the resulting compliance rate is less than 90 percent for Transfers into ICU and Transfers from other facilities. The IP should focus on improving performance related to these two risk categories.

18. The Hemodialysis Department at a hospital completes a monthly water culture testing. The results are over the limit for colony-forming unit (CFU)/mL. Which of the following actions should be done first? a. Notify the physician b. Initiate daily water culture testing c. Disinfect the system d. Notify the Infection Control Department

18. C Disinfect the system Rationale: According to the Conditions for Coverage document from Centers for Medicare & Medicaid Services (CMS), product water used to prepare dialysate or concentrates from powder at a dialysis facility, or to process dialyzers for reuse, shall contain a total viable microbial count lower than 200 CFU/mL and an endotoxin concentration lower than 2 EU/mL. Measures must be performed promptly when results exceed the action level or the maximum allowable level. Dialysis may continue when bacteria/endotoxin is found to be at the action level, but retesting and/or disinfection of the system should be performed promptly. Promptly has been defined y CMS regulation as within 48 hours of receiving the report.

19. Which of the following is an example of an effective performance measure? a. A measure that has been developed based on observation of practices in a facility b. A measure that is based on a definition that can easily be understood and applied in a facility c. A measure that demonstrates a return on investment (ROI) d. A measure that takes into consideration customer satisfaction

19. B A measure that is based on a definition that can easily be understood and applied in a facility Rationale: Performance measures should be evidence-based, well-defined, clinically important for patient populations, and broadly applicable in different types of facilities. Selection of performance measures will be based on both external and internal measurement requirements.

20. The Product Evaluation Committee has asked the IP to assess a new preoperative skin preparation product that is less expensive than the one the facility is currently using. In researching the background information on the product, the IP reviews several sources of literature. Which of the following sources provides the best evidence for effectiveness of the product? a. The vendor representative's assurance that the new product has been tested and works at least as well as the product currently being used b. An independent, randomized trial that shows with statistical significance that the new product is comparable to the current product in preventing infections when used as a surgical skin prep c. The vendor-sponsored laboratory research showing that skin flora is reduced by the same magnitude with the new product as with the current product d. The testimonial of another IP whose hospital has been using the new product for 6 months

20. B An independent, randomized trial that shows with statistical significance that the new product is comparable to the current product in preventing infections when used as a surgical skin prep Rationale: In randomized clinical trials (RCTs), the participants are randomly assigned to treatment or control groups to ensure that the allocation is unbiased. The RCT design minimizes bias and provides the best evidence for direct causal relationships between the experimental factor and the outcome. A randomized trial that was conducted by someone with no financial inerest in the product and that was conducted under clinical conditions will provide the best evidence for effectiveness.

21. Which of the following is an example of infectious waste? a. An unused syringe and needle that were discarded after accidentally being dropped on the floor b. A gauze pad with a small amount of blood on it c. Gloves that were worn to administer a Hepatitis B vaccine d. Agar plates used for testing sputum samples in the microbiology lab

21. D Agar plates used for testing sputum samples in the microbiology lab Rationale: Any objects with the potential to have sufficient dose of a pathogen, a portal of entry, and a method of being transmitted are considered to be infectious waste. Bacterial cultures that have amplified potential pathogens may also have sharp edges and should be considered infectious waste. Nonpenetrating objects with minimal blood and body fluid contamination are not infectious waste. Unused sharps are not infectious waste but must be disposed of in a sharps container because of the high risk of puncture injuries and inability to know that the object was not contaminated.

24. Which of the following statements is correct regarding the efficacy of a disinfectant product on a microbe? a. For all disinfectants, the greater the concentration of the disinfectant, the shorter the contact time is for effective cidal activity b. For all disinfectants, the activity of the disinfectant increases as temperature increases c. For all disinfectants, the greater the number of microbes on an object or surface, the greater the amount of time that's needed for a disinfectant to have effective cidal activity d. For all disinfectants, the existence of a biofilm does not affect cidal activity

24. C For all disinfectants, the greater the number of microbes on an object or surface, the greater the amount of time that is needed for a disinfectant to have effective cidal activity Rationale: Without exception, a higher bioburden of organisms on an object or surface will require higher contact time for a disinfectant to kill the organisms. Prior cleaning of a surface or instrument will reduce the bioburden, which will decrease the necessary contact time for the disinfection process.

5. Which of the following scenarios is an example of the preparation phase of the transtheoretical model? a. Auditing reveals that staff compliance with personal protective equipment (PPE) use has increased in the last month b. Data is showing a 96 percent compliance rate for PPE use in the last 10 months c. Staff are discussing strategies to increase PPE use at the monthly staff meeting d. Staff are questioning why they need to wear PPE when caring for patients in isolation

25. C Staff are discussing strategies to increase PPE use at the monthly staff meeting Rationale: The principal concept behind the Transtheoretical Model (or Stage Theory) is readiness. For any given health-associated behavior, people will have diverse orientations to change. Some will be unaware that a particular change is a desirable option, whereas others will have already completed the change but remain at risk of reversing their progress or relapsing. The stages include Precontemplation, Contemplation, Preparation, Action, and Maintenance. The preparation phase is when a person or group of people starts planning for the behavior change.

26. The Director of Facilities informs the IP that water testing of the hospital's hot water tanks and distal sites was positive for Legionella. The IP should consider disinfection of the hospital's water system if: a. There have not had any prior cases of healthcare- associated legionellosis b. The IP conducts prospective surveillance of healthcare-associated legionellosis and there are no cases c. The colonization rate for distal water sites in the hospital is greater than 30 percent d. The hospital Risk Manager asks for the disinfection to be completed

26. C The colonization rate for distal water sites in the hospital is greater than 30 percent Rationale: Legionella pneumophila is a common cause of both community-acquired and healthcare-associated pneumonia. Clinical manifestations are nonspecific, but high ever, diarrhea, and hyponatremia are often distinctive. Infection has been linked to drinking water distribution systems of acute care and extended care facilities. Health departments and public agencies have issued infection prevention guidelines aimed at preventing outbreaks. These guidelines include diagnostic testing for Legionella infection and culturing of the drinking water distribution system. Disinfection of the water distribution system includes superheating and flushing with hyperchlorination as a short-term approach to terminating an outbreak. Copper-silver ionization has been validated for long-term systemic disinfection. Chlorine dioxide is a promising alternative disinfection method. Disinfection of the facility's water system should be considered if there is evidence of a prior case(s) of healthcare-associated legionellosis, greater than 30 percent colonization of distal sites in the water system, or prospective surveillance for legionellosis detects a healthcare-associated case.

27. The IP is planning an educational program for the nursing staff. The IP knows that in order to increase retention and motivate the nurses to change practice behaviors, she should do which of the following? 1) Assume a facilitator role 2) Utilize monologues to provide critical information 3) Incorporate an interactive approach 4) Use a computer-based learning module a. 1, 3 b. 1, 4 c. 2, 3 d. 1, 2

27. A 1, 3 Rationale: To increase retention and motivate the learner to change practice behaviors, the educator should assume a facilitator role, limit monologues or lectures, and opt for more interactive classroom approaches. A rule of thumb for the active/passive ratio is a minimum of 60/40. This mix of activities and presentation methods will also help to hold the adult learner's interest. Providing a safe, low-risk, nonthreatening learning environment can facilitate class interaction.

29. The purpose of the annual infection prevention program risk assessment is to: a. Determine goals and objectives for the following year b. Describe support requirements of the program c. Outline the achievements and activities of the program d. Document the facility's risks of infection

29. A Determine goals and objectives for the following year Rationale: An annual risk assessment must be performed to determine goals and objectives for the infection prevention program. These should be based on the institution's strategic goals and institutional data and findings fom the previous year's activities. Infection prevention resources and data systems needs should be evaluated in the context of these goals and objectives.

52. Diluted bleach solutions stored in an open container must be changed every: a. 30 days b. 28 days c. 24 hours d. 12 hours

52. C 24 hours Rationale: If a diluted bleach solution is stored in an open container, the chlorine rapidly dissipates and therefore must be prepared daily. When stored in a spray or wash bottle or in a closed, brown opaque container, the bleach solution remains stable for 30 days and will retain 50 percent of its initial value.

71. Decontamination is the process by which an item is: a. Cleaned of all soil and germs b. Rendered free from all pathogens and infectious organisms c. Sterilized and ready for reuse d. Rendered safe for handling without protective attire

71. D Rendered safe for handling without protective attire Rationale: Decontamination renders an area, device, item, or material safe to handle (i.e., safe in the context of being reasonably free from a risk of disease transmission). The primary objective is to reduce the level of microbial contamination so that infection transmission is eliminated.

31. The Director of Infection Prevention and Control has just received information about a bioterrorist threat in her county with an agent spread primarily by contaminated food/water. Which of the following agents should she include in her emergency response plan? a. Tularemia b. Vibrio cholerae c. Q fever d. Brucellosis

31. B Vibrio cholerae Rationale: Bioterrorism refers to the use of biological agents on civilian or military populations, animals, or crops. There are a broad range of potential bioterrorism agents, including bacteria, viruses, and toxins (of microbial, plant, or animal origin). Common characteristics of this diverse group of agents include (1) the ability to be dispersed in aerosols of 1 to 5 μm particles, which can penetrate the distal bronchioles; (2) the ability to deliver these aerosols with simple technology; (3) the feasibility of these agents, if delivered from a line source (e.g., an airplane) upwind from the target, to infect large numbers of the population; and (4) the ability to spread infection, disease, panic, and fear. The U.S. public health system and primary healthcare providers must be prepared to address various biological agents, including pathogens that are rarely seen in the United States. The CDC classifies high-priority agentsas Category A. This category includes organisms that pose a risk to national security because they have the following characteristics:

32. A patient is admitted to rule out TB. All of the following would be noted as a risk factor for transmission of infection to the staff except: a. Improper N95 respirator use by the staff b. Surgical mask placed on the patient during transport c. Acid-fast bacilli (AFB) smear positive sputum culture from the patient d. Patient placed in a regular exam room in the emergency room

32. B Surgical mask placed on the patient during transport Rationale: The risk of infection transmission is related to the number of organisms that are aerosolized. Increased risk of infection is associated with patients with cavitary disease, smear-positive sputum, or a cough. Risk is also increased if patients are placed in poorly or improperly ventilated rooms. The emergency room (ER) and clinic areas should have plans for appropriately isolating patients with suspected TB who are being seen as outpatients. These plans should include details on placing the patient in a separate area from other patients and placing a surgical mask on him or her until appropriate isolation can be arranged.

33. In the event of a disaster with a prolonged power outage, how should you prioritize use of food stored in the facility? a. You should continue to use food from a variety of refrigerators and freezers as usual b. You should prepare food that is in unpowered refrigerators first c. You should prepare food that is in unpowered freezers first d. You should immediately use disaster reserve supplies

33. B You should prepare food that is in unpowered refrigerators first Rationale: In the event of an emergency where there is a power outage, the use of food should be prioritized to maximize supplies and to prevent the occurrence of foodborne illnesses. Food in an unpowered refrigerator should be used firt because the temperature in this appliance will drop most rapidly below the critical temperature for food safety. If food is held at room temperature for more than two hours or above 90°F for more than an hour, it should be discarded because of the risk of microbial growth.

34. Which of the following diseases are preventable by immunization? 1) Diphtheria 2) Varicella 3) Pertussis 4) Cytomegalovirus a. 1, 2, 3 b. 1, 2, 4 c. 1, 4, 5 d. 2, 4, 5

34. A 1, 2, 3 Rationale: The ACIP immunization schedule for HCP includes vaccines for diphtheria, varicella, and pertussis. There is no available vaccine for preventing congenital (present at birth) CMV disease. However, a few CMV vaccines are being tested in humans, including live attenuated (weakened) virus vaccines and vaccines that contain only pieces of the virus. The Institute of Medicine has ranked the development of a CMV vaccine as a highest priority because of the lives it would save and the disabilities it would prevent.

35. According to the Centers for Disease Control and Prevention (CDC) and the Advisory Committee on Immunization Practices (ACIP), which of the following groups should receive the Hepatitis B vaccine? 1) Unvaccinated adults receiving chemotherapy 2) Residents and staff of facilities for developmentally delayed disabled persons 3) People with more than one sexual partner 4) Unvaccinated adults <60 years of age with diabetes mellitus a. 1, 2, 3 b. 2, 3, 4 c. 1, 3, 4 d. 1, 2, 4

35. B 2, 3, 4 Rationale: Hepatitis B vaccination is the most effective measure to prevent HBV infection and its consequences, including cirrhosis of the liver, liver cancer, liver failure, and death. In adults, ongoing HBV transmission occurs primarily among unvaccinated persons with behavioral risks for HBV transmission, such as heterosexuals with multiple sex partners and men who have sex with men. Developmentally disabled persons in residential and nonresidential facilities also have had high rates of HBV infection, but the prevalence of infection has declined since the implementation of routine Hepatitis B vaccination in these settings. However, since Hepatitis B surface antigen-positive persons reside in such facilities, clients and staff continue to be at risk for infection. In response to multiple outbreaks of HBV among persons receiving assisted blood glucose monitoring, it is now recommended that all previously unvaccinated adults aged 19 through 59 years with diabetes mellitus (type 1 and type 2) be vaccinated against Hepatitis B as soon as possible after a diagnosis of diabetes is made.

51. What recommendation should an IP give regarding mask use by a staff member who has documented immunity to varicella (chickenpox) while caring for a patient who is on Airborne Precautions for varicella (chickenpox)? a. Wear an N95 respirator whenever entering the room b. Wear a regular surgical mask whenever entering the room c. No mask is needed d. The patient should don a surgical mask

51. C No mask is needed Rationale: According to the HICPAC 2007 Guideline for Isolation Precautions, there are no recommendations for HCP who are immune to measles and chickenpox (varicella) to wear respiratory PPE. There are also no recommendations for susceptible HCP to wear a surgical mask versus an N95 respirator when caring for patients with measles or chickenpox (varicella).

36. When performing an audit of the Endoscopy Department, the IP observes the following practices. Which one is cause for concern? a. Single-use brushes are used to clean the scopes b. Equipment is immersed in enzymatic cleaner that is discarded every 24 hours c. Leak testing performed on scopes after each use d. Scopes stored vertically in a closed cabinet

36. B Equipment is immersed in enzymatic cleaner that is discarded every 24 hours Rationale: Infection prevention is dependent on the education, training, and skill of the practitioner, the integrity of the device (ensuring that the equipment is free of defects), and strict adherence to reprocessing protocols. Flexible endoscopes are considered semicritical devices because they come into contact with mucous membranes but do not usually enter sterile tissue or the vascular system. Endoscopes should, at a minimum, receive high-level disinfection. Enzymatic detergents must be discarded after each use, as these products are not microbicidal and will not retard microbial growth.

37. While conducting rounds in the Intensive Care Unit (ICU), the IP is approached by a nurse who wants to know what the best practice standards are for drawing blood cultures from an existing central line. The correct response is: a. Drawing blood cultures from an established central line reduces the contamination rate b. Blood samples should be obtained in pairs (two sets) from different peripheral sites c. The site is unimportant as long as the hub is cleaned thoroughly d. The tip of the catheter is also cultured if bacteremia is suspected

37. B Blood samples should be obtained in pairs (two sets) from different peripheral sites Rationale: Specimen collection from central catheters is not recommended due to the possibility of intraluminal bacterial contamination of the device. Percutaneous venipuncture from two separate sites is preferred.

38. Which of the following organisms is most likely to be associated with contaminated disinfectants? a. Pseudomonas species b. Hepatitis C c. Clostridium difficil d. Cryptosporidium parvum

38. A Pseudomonas species Rationale: Although disinfectants are formulated to kill microbes, they can become contaminated during use and can spread the contaminating microbe in the environment. Species of the genus Pseudomonas have been isolated in over 80 percent of contaminated products. To prevent contamination of disinfectants, the products should not be diluted unless specified y the manufacturer, workers who prepare and use disinfectant solutions should be trained on the common ways that these solutions become contaminated, and disinfectant solutions should be stored per the manufacturer's recommendations.

39. In 2013, 3,254 persons died of all causes in a large metropolitan area with a population of 1.8 million. What was the crude mortality rate?a. 18 per 100,000 b. 18 per 1,000 c. 180 per 1,000 d. 180 per 100,000

39. D 180 per 100,000 Rationale: A mortality rate is the measure of the frequency of death in a defined population during a specified time (usually a ear). The crude mortality rate measures the proportion of the population dying each year from all causes. The cause-specific mortality ate measures mortality from a specified cause or a population. Mortality rate = x/y × k Where:x = The number of people in a defined population during a specified interval of time who (1) die of any cause (crude rate) or (2) die of a specified cause(cause-specific ate)y = Estimated population at midyear (i.e., July); crude rates use 1,000 or 100,000k = Usually an assigned value of 1,000 when calculating crude rates: 100,000 is used for cause-specific atesIn this scenario the crude mortality rate is calculated as 3.254 ÷ 1,800,000 × 100,000 = 180 per 100,000 population.

41. Methods to prevent the transmission of C. difficil infections include: 1) Hand washing with soap and water 2) Environmental cleaning and disinfection 3) Closing the unit for deep cleaning 4) Laundry and waste management controls a. 1, 2 b. 2, 4 c. 1, 3 d. 3, 4

41. A 1, 2 Rationale: C. difficil is a Gram-positive, spore-forming anaerobic bacillus that produces two large toxins—A and B—that cause diarrhea and colitis in susceptible patients whose normal colonic bacterial floa has been disrupted by prior antimicrobial treatment. Measures directed at the interruption of horizontal transmission include barrier precautions (hand hygiene/washing, gloving, isolation, and cohorting), and environmental cleaning and disinfection

43. Event-related storage of sterile items allows packaged, sterile items to be used any time after processing provided that: 1) The expiration date has not passed 2) The packing wrapper is intact 3) The item has not gotten wet 4) The storage area is well-ventilated, dry, and free of dust and insects a. 1 b. 1, 2, 3 c. 2, 3, 4 d. 1, 2, 3, 4

43. C 2, 3, 4 Rationale: Event-related storage of sterile items allows for items to be used at any time after processing, provided the sterile packaging has not been compromised. Items must be stored in a dry, well-ventilated, dust-free, insect-free, and temperature controlled area. Packages must be inspected before use to ensure that the wrapping is still intact, with no tears, punctures, or evidence of water intrusion.

44. The Infection Prevention Manager is analyzing the department's overall performance. The ratio of output to input will produce a measure of the department's: a. Productivity b. Effectiveness c. Quality control d. Throughput

44. A Productivity Rationale: Productivity is the ratio of output to inputs in production. It is an average measure of the efficiency of production. Efficiency of production means production's capability to create incomes, which are measured by subtracting real input value from real output value.

45. There is a flu outbeak in a community, and the IP at the local hospital is interested in accurately identifying infected patients as quickly as possible so that they can be placed on Isolation Precautions. The IP has a choice between two rapid flu tests. Test A has a sensitivity of 98 percent and a specificity of 8 percent. Test B has a sensitivity of 92 percent and a specificit of 89 percent. Which test will be most accurate in correctly identifying patients who are infected with flu a. Test A, because it has higher sensitivity b. Test A, because it has a higher predictive value negative c. Test B, because it has higher specificity d. Test B, because it has higher predictive value negative

45. A Test A, because it has higher sensitivity Rationale: Test A is the best choice. It will correctly identify 98 percent of people who have flu because it has 98 percent sensitivity. The specificity of Test A is 85 percent, so 15 percent of patients who do not have flu will be identified as positive (15 percent false positives [FPs]). In this case the IP is most interested in isolating infected patients, so the false-positive rate is an acceptable trade-off in this example.

46. Of the recommendations for reprocessing flexible gastrointestinal endoscopes listed below, the one that is most strongly supported by well-designed experimental, clinical, or epidemiologic studies is: a. Cleaning of endoscopes is essential before manual or automated disinfection b. Brushes used to clean endoscopes should be thoroughly cleaned and disinfected or sterilized between uses c. Ultrasonic cleaning of reusable endoscopic components should be used to remove soil and organic material d. Routine environmental microbiological testing of endoscopes should be carried out

46. A Cleaning of endoscopes is essential before manual or automated disinfection Rationale: Cleaning of endoscopes before manual or automated disinfection is a category 1A recommendation by the CDC's HICPAC and is critical for prevention of endoscopy-related HAIs. There is no strong evidence to show that cleaning and disinfecting brushes or that ultrasonic cleaning will prevent endoscopy-related HAIs. These are both Category II recommendations. There is no recommendation for the use of routine environmental microbiological testing of endoscopes

48. When a test has a higher specificity than sensitivity, it means the test: a. Will be more accurate when predicting who is ill b. A negative result will be more accurate than a positive c. A positive result will be more accurate than a negative d. It should only be done as a secondary testing procedure to rule out disease

48. B A negative result will be more accurate than a positive Rationale: Sensitivity (also called the true positive rate) measures the proportion of actual positives that are correctly identified as such (e.g., the percentage of sick people who are correctly identified as having the condition). Specificity sometimes called the true negative rate) measures the proportion of negatives that are correctly identified as such (e.g., the percentage of healthy people who are correctly identified as not having the condition).Specificity = TN/(TN+FPTherefore, a test with 100 percent specificity orrectly identifies all ptients without the disease.Reference: APIC Text,

49. A healthcare worker has called the IP to ask the reasoning behind the policy that prohibits employees who have patient contact from wearing artificial fingernails. Which of the following reasons would she give to this employee? a. There is strong evidence that artificial nails ae associated with HAIs b. HCP with artificial nails ae more likely to have more Gram-negative bacteria under their nails than HCP with natural nails c. Artificial nails often puncture gloves, exposing the healthcare worker to blood and body fluid d. HCP with artificial nails do not perform hand hygiene as often as HCP with natural nails

49. B HCP with artificial nails ae more likely to have more Gram-negative bacteria under their nails than HCP with natural nails Rationale: Whether artificial nails contribute to the spread of HAIs is unknown. However, HCP with artificial nails ae more likely to harbor Gram-negative organisms on their fingertips than ae those with natural nails. This holds true both before and after hand washing.

50. Managing infection prevention outcomes, analyzing variance trends, and evaluating corrective actions taken to reduce infection risks are components of: a. Case management b. Utilization review c. Performance improvement d. Medical review

50. C Performance improvement Rationale: Performance improvement is measuring the output of a particular process or procedure, then modifying the process or procedure to increase the output, increase efficiency, or increase the effectiveness of the process or procedure. Performance improvement is an ongoing cycle that focuses on patient clinical outcomes and customer satisfaction and service. Measuring performance determines program effectiveness and efficiency and whether proactive approaches or retrospective analysis of high-risk processes can further improve the infection prevention program.

53. A patient in Droplet Precautions is scheduled for a procedure in Endoscopy. In addition to following respiratory hygiene and cough etiquette, which of the following is the most appropriate method to reduce the transmission of infectious organisms during transportation of the patient? a. Instruct patient to wear an N95 respirator b. Cover or contain potentially infectious body fluid c. Place a surgical mask on the patient d. Provide respiratory protection for transport personnel

53. C Place a surgical mask on the patient Rationale: According to the CDC's 2007 Isolation Guidelines, patient transport outside the room should be limited to medically necessary purposes. If the patient must leave the room, instruct the patient to wear a surgical mask and follow respiratory hygiene and cough etiquette. Once the patient is masked, the patient transporter does not need to wear a surgical mask. Notify the receiving department of the Isolation Precautions status.

54. A pregnant environmental services worker, who is nonimmune to varicella, enters the room of a patient with confirmed varicella (chickenpox) before an isolation sign is posted. She spends 6 minutes in the room with the patient (who is not wearing a mask). This exposure happened on January 11. After giving the employee varicella-zoster immune globulin (VZIG), it is determined that that she should be excluded from work. What day can she return to work? a. January 31 b. February 1 c. February 8 d. February 9

54. D February 9 Rationale: The healthcare worker was given VZIG. Normal time off would be from day 10 through day 21 after exposure, but because VZIG was given, it is recommended to keep the employee off through day 28 (able to return on the 29th day after the exposure).

55. This process destroys all forms of microbial life: a. High-level disinfection b. Cleaning c. Sterilization d. Antisepsis

55. C Sterilization Rationale: Sterilization is the term for the process where all microbial life including spores is destroyed. Sterilization may be carried out using steam, hydrogen peroxide gas, ethylene oxide gas, and liquid sterilants.

56. A patient was admitted to the Emergency Department with severe headache, fever, and photophobia. The results of the lumbar puncture are pending. A staff member who cared for the patient is requesting antibiotics because the patient may have meningitis. If meningococcal meningitis is confirmed, chemopophylaxis would be indicated for all of the following individuals except: a. The licensed practical nurse who did not have direct contact with the patient's oral secretions b. The emergency medical technician who performed mouth-to-mouth resuscitation c. The resident who did not wear appropriate PPE during intubation d. The registered nurse who suctioned the patient without wearing a mask

56. A The licensed practical nurse who did not have direct contact with the patient's oral secretions Rationale: HCP without direct exposure to the patient's oral secretions are considered low risk and chemoprophylaxis is not recommended.

57. Which of the following statements about testing for statistical significance is true? a. A p value of 0.05 means that the probability that the observation occurred by chance alone is 1 in 20 b. A p value of 0.05 increases the likelihood of making a Type 2 error c. The size of the p value indicates the power of the results d. The research hypothesis is the basis of significance

57. A A p value of 0.05 means that the probability that the observation occurred by chance alone is 1 in 20 Rationale: The level of significance is the probability value arbitrarily chosen by the researcher as the desired level of probability at which one may feel secure in rejecting the null hypothesis. When using sample data, it is not possible to be absolutely certain that the hypothesis being accepted is true. Therefore, a probability that the finding is due o chance is stated. This probability of rejecting a null hypothesis when it is true is the level of significance or α level. Most researchers use 0.05 (5 percent) or 0.01 (1 percent) values for α to minimize the chances of incorrectly rejecting the null hypothesis. This specified level states a sufficiently small likelihood that the given observation could occur by chance variation alone (e.g., 0.05 or a 1-in-20 chance). The researcher finds the appropriate rejection region for a test statistic at a given α level and rejects the null hypothesis for values of the test statistic that lie beyond the specified value. Simply stated, α level is the level of risk of being wrong that a researcher is willing to take. The p value is commonly compared to α—the specified significace level of the test. A p value of 0.05 indicates that the probability that the observation occurred by chance alone is 0.05 or 1 in 20. That is, a true null hypothesis will be rejected one out of every 20 times.

58. An IP is writing instructional objectives for a learning module on catheter-associated bloodstream infections. He wants to focus on the cognitive level of "Analysis" from Bloom's taxonomy. Which of the following choices contains the verbs he should use to describe his learning objectives? a. Compare, discriminate, and differentiate b. Define, identify, and select c. Discuss, explain, and paraphrase d. Judge, summarize, and recommend

58. A Compare, discriminate, and differentiate Rationale: Bloom's taxonomy was developed in the 1950s and is still used today to categorize ways of learning and thinking in a hierarchical structure. A revised model was developed in the 1990s to better fit educational practices of the 21st century. The "Analysis" level refers to the process of breaking material into constituent parts, determining how the parts relate to one another and to an overall structure or purpose through differentiating, organizing, and attributing. Applicable verbs include analyze, appraise, break down, calculate, categorize, classify, compare, contrast, criticize, derive, diagram, differentiate, discriminate, distinguish, examine, experiment, identify, illustrate, infer, interpret, model, outline, point out, question, relate, select, separate, subdivide, test.

59. During rounds in several patient care areas, the IP discovers several cardboard boxes being used to store patient care supplies. She informs the unit manager that another method of storing supplies must be implemented because: a. The boxes cause clutter in the storeroom b. Cardboard can only be used to store supplies in soiled utility rooms c. The boxes may harbor dust, bacteria, and insects d. The boxes are a fie hazard

59. C The boxes may harbor dust, bacteria, and insects Rationale: Corrugated cardboard boxes are not appropriate as storage units in medical or clean supply rooms because the boxes may harbor dust, bacteria, and small insects that have entered during shipping.

6. Phlebotomists within an organization are complaining that the new blood collection device introduced 6 months ago is difficult o use for blood draws and has resulted in an increase in needlestick injuries (NSIs). The IP is working with Occupational Health to evaluate the problem and would like to compare NSI rates before and after implementation of the device. Which of the following would be the most useful denominator in order to calculate useful data? a. Phlebotomist employee hours at work (full-time equivalents) b. Number of occupied beds (or licensed beds) c. Number of patients (average daily census) d. Number of blood collection devices used or purchased

6. D Number of blood collection devices used or purchased Rationale: The denominator should represent the potential for exposure to sharps. There are many possible denominators that may be used, and each will provide a different view of the sharps injury situation in the facility. A device-based rate can be used to compare needlestick risk from different devices and to evaluate the effectiveness of the product design. Because blood draws may be performed by nurses, phlebotomists, or physicians, total device-associated needlestick injuries since implementation of the new product will provide the most useful data.

60. An IP is evaluating her control chart and notices that several points in a row are above the mean line. This probably indicates: a. The mean is incorrectly calculated b. She should investigate potential sources of special cause variation c. There is common cause variation in her process, and it requires no correction d. She is using the incorrect type of control chart

60. B She should investigate potential sources of special cause variation Rationale: Statistical process control (SPC) is a method used to monitor both processes and outcomes in a systematic and statistically valid manner. It is a decision-making tool that shows when a process is working correctly and when it is not. This information can then be used to improve quality. Control charts show if a process is in control or out of control. There are eight major rules used to detect special cause variation on a control chart:1. Any point above the Upper Control Limit or below the Lower Control Limit2. One of two points above +2 Standard Deviation (SD) or below -2 SD3. Four of fie points above +1 SD or below -1 SD4. Eight consecutive points above or below the CL5. Six consecutive points increasing or decreasing6. Fifteen consecutive points between +1 SD and -1 SD7. Fourteen consecutive points alternating up and down.8. Eight consecutive points above +1 SD and/or below -1 SD

61. Which of the following organisms are most likely to be the cause of outbreaks of healthcare-associated infections due to improper reprocessing of bronchoscopes? 1) Mycobacterium tuberculosis 2) Carbapenem-resistant Enterobacteriaceae 3) Pseudomonas aeruginosa 4) Staphylococcus aureus a. 1, 2, 3, 4 b. 1, 3, 4 c. 1, 3 d. 2, 4

61. C 1, 3 Rationale: Improperly processed bronchoscopes have been associated with outbreaks of infections in healthcare settings. These incidents have been due to inadequate cleaning of the scope, incorrect selection of disinfecting agents, and lapses in following the manufacturer's protocol for cleaning. The organisms that have been most highly associated with improperly processed bronchoscopes are Mycobacterium tuberculosis and Pseudomonas aeruginosa

62. This type of sterilizer works by forcing steam into the chamber from the top and pushing the air in the chamber out the bottom of the chamber: a. Steam sterilizer b. Ethylene oxide sterilizer c. Dynamic air removal steam sterilizer d. Gravity displacement steam sterilizer

62. D Gravity displacement steam sterilizer Rationale: A gravity displacement sterilizer works by admitting steam into the sterilizer chamber. The steam has lower density than the air in the chamber, so the steam initially stays at the top of the chamber and then pushes the air out of a drain vent in the bottom of the chamber.

63. Which of the following organisms is most likely to contaminate a urine specimen if the collection process is not done correctly: a. Staphylococcus aureus b. Pseudomonas aeruginosa c. Candida albicans d. Klebsiella pneumonia

63. C Candida albicans Rationale: Fungi are eukaryotic organisms with cell walls containing chitin, cellulose, or both. Yeasts are unicellular organisms that reproduce by budding and typically, but not always, produce circular, mucoid colonies upon culture. Clinically, the most frequently isolated yeasts are Candida, of which there are more than 80 species. Candida is a normal commensal of the human gastrointestinal and female genital tracts.

64. Which of the following environmental infection prevention measures are recommended for inpatient units that house hematopoietic stem cell transplant (HSCT) patients? 1) Providing sterile linens 2) Prohibiting live plants and dried or fresh flowers in rooms of HSCT patients 3) Avoiding items that collect or trap dust 4) Providing laminar air flow a. 1, 2 b. 3, 4 c. 2, 3 d. 1, 3

64. C 2, 3 Rationale: Infectious complications are a common occurrence among patients receiving HSCT. Preventive measures should emphasize provision of a protective environment, appropriate use of prophylactic anti-infective agents, and meticulous attention to infection prevention practices, such as hand hygiene, device management, and the regulation of visitors. Furnishings and fixtures in patient rooms should be easy to clean, and items that trap or collect dust should be avoided. Flowers/plants or their potting materials may harbor large numbers of Aspergillus spores and other microorganisms and should be restricted from the patient environment.

66. Which of the sterility assurance levels (SAL) listed below is appropriate for critical items? a. zero b. 10-4 c. 10-6 d. 10-8

66. C 10-6 Rationale: The sterility assurance level (SAL) is the probability of a microbe surviving on an item after sterilization. A SAL of 10-6 means that there is at most a 1 in one million chance of an organism having survived. This SAL is considered to be appropriate for critical items.

67. Green cleaning is a new approach to environmental cleaning that aims at reducing harm to human health and the environment while maintaining or improving the hygiene of the healthcare environment. Which of the following green strategies would be appropriate to implement? 1) Replacing floor trippers and finishes that contain heavy metals and asthmagens 2) Substituting disinfectant room cleaners with less toxic detergents 3) Employing carcinogen-free carpet cleaners 4) Introducing high-filtration vacuum cleaners a. 1, 2, 4 b. 2, 3, 4 c. 1, 3, 4 d. 1, 2, 4

67. C 1, 3, 4 Rationale: The effective use of disinfectants is part of a multibarrier strategy to prevent healthcare-associated infections. CDC Guidelines recommend that noncritical medical equipment surfaces should be disinfected with an EPA-registered low- or intermediate-level disinfectant. Current certification programs for green cleaning products and equipment do not cover EPA-registered disinfectants.

68. Which of the following would be evidence of CAUTI in an adult patient with an indwelling urinary catheter in place for more than 2 days?1) Fever greater than 38°C 2) A positive dipstick for leukocyte esterase 3) A positive urine culture with 104 CFU/ml of E. coli 4) A positive urinary catheter tip for E. coli a. 1, 3, 4 b. 1, 2, 4 c. 1, 2, 3 d. 2, 3, 4

68. C 1, 2, 3 Rationale: Virtually all healthcare-associated urinary tract infections are caused by instrumentation of the urinary tract. For patients with an indwelling urinary catheter at the time of specimen collection, NHSN criteria for a symptomatic urinary tract infection (SUTI) include:• Patient had an indwelling urinary catheter in place for > 2 calendar days, with day of device placement being Day 1, and catheter was in place on the date of event etc....

69. Case-control studies are useful for studying: a. Rare outcomes or outcomes that develop over a long time after exposure b. Individuals with and without exposure to a potential risk factor c. Specific xposure incidents d. The availability of a risk exposure

69. A Rare outcomes or outcomes that develop over a long time after exposure Rationale: Case-control studies may be undertaken in a timelier and less-expensive manner than prospective cohort studies because cases may be identified etrospectively, and at least some exposure data are often available through medical record review.

7. Which of the following processes should be used for contaminated endotracheal blades? a. Cleaning followed by high-level disinfection b. Cleaning with chlorhexidine followed by soaking in an enzymatic solution for 20 minutes c. Cleaning followed by ultrasonic washer d. Cleaning followed by alcohol disinfection

7. A Cleaning followed by high-level disinfection Rationale: Semicritical items are those items that will contact mucous membranes or nonintact skin. Respiratory therapy and anesthesia equipment, some endoscopes, laryngoscope blades, esophageal manometry probes, anorectal manometry catheters, and diaphragm fitting rings ae included in this category. These medical devices should be free of all vegetative microorganisms (i.e., mycobacteria, fungi, viruses, bacteria), though small numbers of bacterial spores may be present. Intact mucous membranes, such as those of the lungs or the gastrointestinal tract, generally are resistant to infection by common bacterial spores but are susceptible to other organisms, such as bacteria, mycobacteria, and viruses. Semicritical items minimally require high-level disinfection using chemical disinfectants.

70. The existence of an outbreak is most often determined by: a. The identifiction of more than 10 new cases per week b. An incidence rate that is more than two standard deviations higher than the previous year c. Detection of a cluster of organisms within more than one population. d. An incidence of disease that is clearly in excess of that expected

70. D An incidence of disease that is clearly in excess of that expected Rationale: Outbreaks in healthcare should be suspected when HAIs or adverse events occur above the background rate or when an unusual microbe or adverse event is recognized. Healthcare-associated outbreaks often have multiple causes, but almost all are due to one or more of the following: lapses in infection prevention or clinical practices, colonization or infection of HCP, or defects in or contamination of a product or device, either at the time of production (intrinsic contamination) or during use (extrinsic contamination). Outbreaks in healthcare settings may also be caused by visitors who have, or are harboring, an infectious disease (e.g., influenza or chickenpox).

72. While caring for a patient with suspected or confirmed Ebola if during patient care a partial or total breach in PPE (gloves separate from sleeves leaving exposed skin, a tear develops in an outer glove, a needlestick) occurs, the healthcare worker must: a. Quickly remove PPE to reduce the risk of exposure b. Immediately perform disinfection of gloved hands using an ABHR c. Move immediately to the doffing area to assess the exposure d. Immerse exposed areas with a bleach solution

72. C Move immediately to the doffing area to assess the exposure Rationale: Prior to working with patients with EVD, all healthcare workers must have received repeated training and have demonstrated competency in performing all Ebola-related infection control practices and procedures, and specifically in donning doffing proper PPE. While working in PPE, healthcare workers caring for patients with EVD should have no skin exposed. If during patient care a partial or total breach in PPE (gloves separate from sleeves leaving exposed skin, a tear develops in an outer glove, a needlestick) occurs, the healthcare worker must move immediately to the doffing aea to assess the exposure. Implement the facility exposure plan, if indicated by assessment.

73. While making rounds on a nursing unit, the IP encounters a nurse who has just incurred a needlestick after giving an injection to a patient. The nurse tells the IP that the patient's recent laboratory results indicate that he has Hepatitis B, and the nurse has not completed her Hepatitis B vaccination series. She requests the IP's advice. In reviewing the patient's record, the IP finds tht the patient's antibody to Hepatitis B is positive and his Hepatitis B surface antigen (HBsAg) is negative. The IP's explanation of the patient's laboratory results indicates which of the following? a. The patient is in the early incubation period of the disease and has a low probability of transmitting Hepatitis B; the nurse should report to Occupational Health b. The patient had Hepatitis B in the past and does not have active disease now; the nurse should complete her Hepatitis B series as scheduled, but no additional treatment is needed c. The results indicate a possible error in the results and another blood sample should be submitted d. The patient does have Hepatitis B but the disease is resolving; the nurse should report to Occupational Health

73. B The patient had Hepatitis B in the past and does not have active disease now; the nurse should complete her Hepatitis B series as scheduled, but no additional treatment is needed Rationale: HBsAg is a protein on the surface of HBV; it can be detected in high levels in serum during acute or chronic HBV infection. The presence of HBsAg indicates that the person is infectious. The presence of Hepatitis B surface antibody (anti-HBs) is generally interpreted as indicating recovery and immunity from HBV infection. This patient's surface antibody is positive and the surface antigen is negative, indicating that he is immune due to a natural infection. The nurse does not have to take any action.

74. Measures of dispersion to consider for surveillance reporting include which of the following? a. Rates b. Ratios and proportions c. Percentiles d. Range and standard deviation

74. D Range and standard deviation Rationale: Measures of dispersion are important for describing the spread of the data or its variation around a central value (mean). Commonly used measures of dispersion are the range, deviation, variance, and standard deviation. The range is the difference between the highest value in a data set and the lowest value. The standard deviation is a measure of how much the data are scattered around the mean and is the square root of the sample variance.

75. Infection of short-term intravenous devices is associated with all of the following except: a. Percutaneous transmission during insertion b. Colonization of the catheter hub c. Extraluminal transmission d. Contamination of intravenous (IV) fluid

75. B Colonization of the catheter hub Rationale: With short-term intravenous devices (IVDs) (in place fewer than 10 days), such as peripheral IV catheters, arterial catheters, and noncuffed, nontunneled CVCs, most device-associated bloodstream infections (BSIs) are of cutaneous origin, from the insertion site, and gain access extraluminally, occasionally intraluminally. In contrast, contamination of the catheter hub and luminal fluid is the pedominant mode of BSI with long-term IVDs (e.g., in place more than 10 days), such as cuffed Hickman- and Broviac-type catheters, subcutaneous central ports, and PICCs.

76. The IP is asked to review with a group of staff nurses how to interpret antibiotic susceptibility tests. The susceptibility test that allows a determination of the least amount of antibiotic per milliliter that impedes the growth of an organism is known as a: a. Minimum inhibitory concentration b. Kirby-Bauer disc method c. Minimum bactericidal concentration d. Serum-cidal level

76. A Minimum inhibitory concentration Rationale: Minimum inhibitory concentration is the lowest concentration of an antimicrobial that will inhibit the visible growth of a microorganism after overnight incubation. Minimum inhibitory concentrations are important in diagnostic laboratories to confirm resistance of microorganisms to an antimicrobial agent and to monitor the activity of new antimicrobial agents.

77. Under what circumstances should human immunodeficieny virus (HIV)-infected HCP be placed under work restrictions? 1) When viral burden measurements of <5 × 102 GE/mL for HIV infection and techniques for eliminating transmission risks cannot be identifie 2) No work restrictions necessary if Standard Precautions are followed 3) When required by state or regional regulations 4) Whenever performing exposure-prone, noninvasive procedures 5) After counsel from an expert review panel is sought a. 2, 5 b. 1, 3 c. 3, 5 d. 4, 5

77. C 3, 5 Rationale: The CDC recommends that exposure-prone invasive procedures should not be performed by HCP with HIV until counsel from an expert review panel is sought. Risks during noninvasive procedures would be minimized with the use of Standard Precautions, but this may not be true for high-risk invasive procedures. An expert review panel can make these determinations and recommendations. State or regional recommendations must be followed. In addition to CDC, the Society for Healthcare Epidemiology of America and the Association for Professionals in Infection Control and Epidemiology provide guidance on this issue.

78. Poor planning during a construction project can lead to an increase risk of infection related to: 1) Construction delays 2) Compromised air quality 3) Contaminated water 4) Increase in construction-related traffi a. 1, 3 b. 2, 4 c. 2, 3 d. 1, 4

78. C 2, 3 Rationale: Insufficient planning can lead o compromised air quality and potential for continued environmental contamination from fungi (e.g., Aspergillus spp.) or water contaminated with water-associated microorganisms (e.g., Legionella spp.) during construction or renovation.

79. The following factors should be considered when preparing the environment for an educational program: 1) Providing an atmosphere of mutual respect and support 2) Establishing a comfortable environment conducive to learning 3) Encouraging interaction by arranging desks in straight rows 4) Having a person control and troubleshoot the environment as needed a. 1, 2, 4 b. 2, 3, 4 c. 1, 3, 4 d. 1, 2, 3

79. A 1, 2, 4 Rationale: One of the most important roles of the educator is to provide an atmosphere of mutual respect, as well as one that is friendly, informal, and supportive. Eye contact, addressing students by name, listening without interrupting, and acknowledging the validity of problems or opinions expressed are characteristics of an effective educator. The educator must also take steps to create an environment that is comfortable and conducive to learning. The learning space should be private and congenial with careful consideration to seating, room temperature, and lighting. There should be a contact person at the learning site to troubleshoot any facility or technical problems should they occur. The traditional classroom setup with straight rows of desks does not promote interaction.

80. Which of the following is an example of syndromic surveillance? 1) Analyzing Emergency Department records for reports of influenza-lie illness (ILI) 2) Monitoring sentinel chickens in the community for antibodies to arboviruses 3) Tracking all laboratory orders for respiratory cultures 4) Monitoring over-the-counter drug sales for cough medicine a. 2, 3, 4 b. 1, 2, 4 c. 1, 2, 3 d. 1, 3, 4

80. D 1, 3, 4 Rationale: Syndromic surveillance is used for early detection of outbreaks; to follow the size, spread, and tempo of outbreaks; to monitor disease trends; and to provide reassurance that an outbreak has not occurred. Syndromic surveillance systems use existing health data in real time to provide immediate analysis and feedback to those charged with investigation and follow-up of potential outbreaks. The fundamental objective of syndromic surveillance is to identify illness clusters early, before diagnoses are confirmed and reported to public health agencies, and to mobilize a rapid response, thereby reducing morbidity and mortality. Answers 1, 3, and 4 are all examples of syndromic surveillance. The monitoring of chickens for antibodies to arboviruses is an example of sentinel surveillance, which is an alternative to population-based surveillance and involves collecting data from a sample of reporting sites (sentinel sites).

81. The IP is performing the annual evaluation of the infection prevention and control program. Components of this document should include: 1) The achievements and activities of the program 2) Results from the latest accreditation survey 3) Stress the value of the program to the organization 4) Satisfy the legal requirements for reporting infections a. 1, 2 b. 2, 3 c. 1, 3 d. 3, 4

81. C 1, 3 Rationale: An annual evaluation of the infection prevention program is important to outline achievements and activities of the program and describe support requirements. The value of the infection prevention program to the organization should be emphasized, along with patient outcomes and cost savings.

82. A bacterium that is decolorized with alcohol during a Gram stain and retains the counterstain is: a. Gram-positive b. Gram-negative c. Stained purpled. Acid-fast positive

82. B Gram-negative Rationale: In the Gram stain technique, bacteria are fixed on a slide and a primary stain, crystal violet, is added to the slide. All bacteria will stain purple at this point regardless of their Gram-reaction category. Iodine is added to fix the crystal violet stain to the peptidoglycan layer of the cell wall of the bacteria, and then alcohol is used as a decolorizing agent. Gram-positive bacteria have a thick peptidoglycan layer that will retain the crystal violet stain even after alcohol is added, whereas Gram-negative bacteria have a lipopolysaccharide layer and a thin peptidoglycan layer. The lipopolysaccharide layer will be dissolved with alcohol and the peptidoglycan layer will be decolorized. The counterstain, safranin, will then stain the decolorized Gram-negative bacteria red; the Gram-positive bacteria will continue to appear purple.

83. Which type of room would be the most appropriate to place a patient who is immunocompromised and has disseminated herpes zoster disease? a. A shared, standard room b. A private, standard room c. An airborne infection isolation (AII) room d. A protective environment room

83. C An airborne infection isolation (AII) room Rationale: Herpes zoster, also known as zoster and shingles, is caused by the reactivation of the varicella-zoster virus (VZV), the same virus that causes varicella (chickenpox). People with herpes zoster most commonly have a rash in one or two adjacent dermatomes (localized zoster). The rash most commonly appears on the trunk along a thoracic dermatome. The rash does not usually cross the body's midline. However, approximately 20 percent of people have rash that overlaps adjacent dermatomes. Less commonly, the rash can be more widespread and affect three or more dermatomes. This condition is called disseminated zoster. This generally occurs only in people with compromised immune systems. Disseminated herpes zoster can be transmitted by an airborne route, so the patient needs to be placed in an airborne isolation room with negative airflw for the duration of the illness.

84. Seventy-fie patients were admitted to the Medical-Surgical ICU. Forty were on the surgical service and 35 were on the medical service. Fifteen patients developed a healthcare-associated MRSA infection. Nine of the patients with MRSA infection were on the surgical service. There were 230 patient days in the ICU for the surgical patients in January, and 325 patient days for medical patients. What was the overall MRSA attack rate? a. 20 percent b. 2 percent c. 53 percent d. 5 percent

84. A 20 percent Rationale: An attack rate is a special form of incidence rate. It is not truly a rate, but a proportion. It is the proportion of persons at risk who become infected over an entire period of exposure or a measure of the risk or probability of becoming a case. It is usually expressed as a percentage and is used almost exclusively for epidemics or outbreaks of disease where a specific population is exposed to a disease for a limited time. The attack rate equals the number of new cases of disease (for a specified time period) divided y the population at risk for the same time period multiplied by 100. Attack rate is the same as incidence rate, except that attack rates are always expressed as cases per 100 populations or as a percentage. The attack rate for this scenario is calculated as follows: 15 ÷ 75 × 100= 20 percent.

85. All of the following are examples of risk-adjusted stratification except: a. CLABSI rates by birth weight in the NICU b. Needlestick injuries by profession c. CAUTI rate for the ICU d. CLABSI rates by type of line

85. C CAUTI rate for the ICU Rationale: Stratifiction is a form of risk adjustment that involves classifying data into subgroups based on one or more characteristics, variables, or other categories. For example, a measure's population might be stratified by gender before calculating rates, resulting in separate rates for males and females. In infection prevention, it is common to stratify infants by birth weight when assessing infections and infection risk or to stratify sharps injuries by time of day, role, and unit. Each subgroup becomes a separate denominator (population of interest), with the numerator event of interest the same for the subgroups; separate rates are then calculated for each subgroup.

86. The IP hears that one of the nursing units in his facility will soon be renovated. After verifying the information, he contacts the newly hired Director of Design and Construction to explain how important it is for the IP to be included in the planning and design of all renovation projects. Which of the following is the most compelling argument? a. The IP supplies necessary maintenance for critical utility systems that deliver ventilation and water to patient care areas b. The IP provides essential input into preventing hazardous risks to patients, HCP, and visitors during design and construction projects c. The IP will ensure compliance with various compliance-, regulatory standard-, and guideline-setting agencies d. The IP is responsible for facilitating the transport and approval for disposal of waste materials

86. B The IP provides essential input into preventing hazardous risks to patients, HCP, and visitors during design and construction projects Rationale: A key element that IPs bring to the construction and renovation process is creating an environment of care that supports prevention of infection and promotes safety of patients and personnel.

87. Viral infections are difficult o treat because: a. Viruses can suspend their metabolism in the presence of antivirals b. Viruses use the host's cells to replicate c. Synthesis of enzymes that inactivate the drug d. Viruses may block viral mRNA transcription

87. B Viruses use the host's cells to replicate Rationale: It is difficult o designing safe and effective antiviral drugs because viruses use the host's cells to replicate. This makes it challenging to find targets for the drug that would interfere with the virus without also harming the host organism's cells.

88. The Infection and Prevention Control Team has adopted the reduction of healthcare-associated MRSA as an annual goal. It has implemented improved processes and plans to monitor the effectiveness of these processes. Which of the following performance improvement tools will assist in identifying effectiveness? a. Affinity diagram b. Run chart c. Process flow chart d. Pareto chart

88. B Run chart Rationale: Run charts are useful for identifying variations and trends, especially when assumptions for construction of control charts are not met. Run charts display observed data that can reveal trends or patterns over a specified period of time. They can be used with any type of data (discrete, continuous, etc.), and do not use any statistical calculations aside from measures of central tendency. They require at least 20 data points for reliability.

89. A patient is admitted with skin and soft tissue injury from a cat bite. The most likely organism involved in the infection would be: a. Pasteurella b. Pseudomonas c. E. coli d. Mycobacterium

89. A Pasteurella Rationale: Cat bites are more likely to become infected (28 to 80 percent) than dog bites (3 to 18 percent), and Pasteurella spp. (primarily P. multocida) is the most common isolate from dogs as well as cats. Pasteurella spp. is highly pathogenic, inducing progressive soft tissue infections with a typically rapid onset (often < 24 hours). Streptococci, staphylococci, Moraxella spp., Corynebacterium spp., and Neisseria spp. were the next most common aerobic isolates derived from cat bites.

90. The OR notifies the IP that a patient is scheduled for a brain biopsy. After reviewing the results of diagnostic tests, possible diagnosis, and the reason for biopsy, the IP assesses the patient to be high risk for Creutzfeldt-Jakob disease (CJD). The next step is to: a. Determine the instruments to be used and the processing to be done b. Notify the staff to quarantine the patient after the procedure c. Advise the OR staff to disinfect the instruments in the OR d. Incinerate all equipment used to perform the biopsy

90. A Determine the instruments to be used and the processing to be done Rationale: The OR, Sterile Processing Department, and the IP should meet to review the policy and recommendations for disinfecting and sterilization of the instruments as outlined by the World Health Organization (WHO) and the CDC.

91. The CDC lists 18 drug-resistant threats to the United States. These threats are categorized based on level of concern: urgent, serious, and concerning. Which of the following belong to the "Urgent" threat category? 1) Neisseria gonorrhea 2) Vancomycin-resistant Staphylococcus aureus 3) Drug-resistant Tuberculosis 4) Clostridium difficil a. 1, 2 b. 2, 3 c. 1, 4 d. 3, 4

91. C 1, 4 Rationale: Infections classified as urgent threats include carbapenem-resistant Enterobacteriaceae (CRE), drug-resistant gonorrhea, and Clostridium difficil, a serious diarrheal infection usually associated with antibiotic use. These are high-consequence threats due to the significant risks identified aoss several criteria. These threats have the potential to become widespread and require urgent public health attention to identify infections and limit transmission.

92. The Infection Prevention Manager must apply principles of conflict resolution to resolve growing tension about how best to apply limited department resources in the coming year. For maximum success, the IP manager should use all of the following approaches except: a. Active listening b. Accept responsibility c. Use indirect communication d. Identify points of agreement

92. C Use indirect communication Rationale: Conflict resolution refers to the methods and processes involved in facilitating the peaceful ending of conflict and preventing retribution. A wide range of methods and procedures for addressing conflict exist and include negotiation, mediation, diplomacy, and creative peace building. Principles of conflict resolution include:• Use direct communication• Listen actively• Think before reacting• Attack the problem, don't make it personal• Accept responsibility• Look for common interests• Focus on the future

93. The IP is designing a new hand hygiene intervention for HCP based on the Health Belief Model. According to that model, which of the following is/are examples of "modifying factors" in the hand hygiene intervention? a. The ages and genders of the people for whom the intervention is being designed b. Signs posted in the unit that remind HCP to perform hand hygiene c. The perception of how much time it will take HCP to perform hand hygiene compared to the potential for hand hygiene to prevent infection in patients d. The belief among the target audience that they will be able to comply with hand hygiene 100 percent of the time

93. A The ages and genders of the people for whom the intervention in being designed Rationale: The health belief model (HBM) is the oldest theory specifically developed to understand and predict health-associated behavior. An IP can use this model as a theoretical framework to motivate and influence infection prevention behaviors of HCP. The HBM includes key components of perceived susceptibility, perceived severity, perceived benefits, and perceived barriers. According to the HBM, HCP would adhere to hand hygiene if they believed that they were susceptible to infection if they neglected to wash their hands. Education on infection prevention measures may influence HCP's perceived risk of contracting and spreading infection. HCP may perceive severity by understanding the serious consequences of infection caused by poor hand hygiene compliance, such as prolonged hospital stay, expensive medical cost, and increased morbidity. HCP may perceive benefits regarding the effectiveness of hand hygiene practice when it comes to decreasing infection among patients and thereby decreasing HCP's heavy workload.

94. A patient in your facility has been diagnosed with naturally-acquired inhalational Anthrax, and Environmental Services need to know how to terminally clean the patient's room. Which protocol should they follow? a. They should clean the room with a protocol similar to the one used for patients with C. difficil b. They should only use an EPA-registered disinfectant with proven effectiveness against Anthrax c. The should call the Centers for Disease Control and Prevention to come clean the room d. They should use the standard terminal cleaning protocol

94. D They should use the standard terminal cleaning protocol Rationale: Anthrax is an illness caused by Bacillus anthracis, a spore-forming bacterium. Infection with Bacillus anthracis can be acquired through exposure to contaminated soil, water, or animals, including imported animal skin drums. It can also be acquired as a consequence of a bioterrorism event, where the spore form of the agent might be released into the environment. In the case of a naturally-acquired case of anthrax, there is no person-to-person transmission of the agent and no environmental contamination of the patient room with the spores. Therefore, no special cleaning protocol is needed in this case. However, a bioterrorism event with release of spores in the facility would warrant higher level decontamination and would warrant higher level decontamination and cleaning.

95. A healthcare facility is experiencing its first case of carbapenem-resistant Enterobacteriaceae (CRE) infection. The IP plans to intensify the facility's MDRO control efforts and is implementing an active surveillance culture (ASC) program for CRE. Which of the following strategies should the IP implement? 1) Screen all patients for CRE on admission 2) Place all colonized or infected CRE patients on Contact Precautions 3) Minimize invasive devices 4) Communicate results to healthcare providers a. 1, 2, 3 b. 2, 3, 4 c. 1, 3, 4 d. 1, 2, 4

95. B 2, 3, 4 Rationale: The emergence and dissemination of carbapenem resistance among Enterobacteriaceae in the United States represent a serious threat to public health. These organisms are associated with high mortality rates and have the potential to spread widely. Clinicians play a critical role in slowing the spread of CRE. Rapidly identifying patients colonized or infected with these organisms and placing them in Contact Precautions when appropriate, using antibiotics wisely, and minimizing device use are all important parts of preventing CRE transmission. Rather than screening all patients on admission, the CDC recommends focusing surveillance testing on patients admitted to certain high-risk settings (e.g., ICUs, long-term acute care) or specific ptients (i.e., patients with risk factors, patients admitted from high-risk settings like long-term acute care or transferred from areas with high CRE prevalence).

96. A patient is receiving eye drops to treat conjunctivitis. The drops are only available in a multi-dose vials. Which of the following is/are acceptable practices for this use of this medication? 1) This vial of medication may not be used on multiple patients 2) This vial of medication may be used on multiple patients if it has not come into contact with tears or the conjunctiva 3) This vial of medication may be used on multiple patients if it is used within 28 days of being opened 4) This vial of medication should be stored away from the patient care area a. 1 b. 1, 4 c. 2, 3 d. 2, 3, 4

96. A 1 Rationale: Multidose eye drops or creams are not acceptable for reuse on other patients if they have been used on a patient with an infectious disease of the eye. If the patient does not have an infectious disease of the eye, then the multidose vial may be used for other patients provided that it has not come into contact with the conjunctiva or tears of a patient. This medication must be used within 28 days of being opened or per the manufacturer's recommendations. Multidose medications that will be used on more than one patient must be stored away from the patient care area.

97. The IP has just reviewed the current public health recommendations concerning influenza vaccines before developing an educational program for employees. The report indicates that the most important problem in developing a long-term vaccine for influenza is a. Potential toxicity of the vaccine if the dosage is increased b. Lack of potency c. Antigenic drift of the viruses d. Short storage life of the vaccine

97. C Antigenic drift of the viruses Rationale: Antigenic drift refers to small changes in the influenza virus that happen continually over time. Antigenic drift produces new virus strains that may not be recognized by the body's immune system. When such a change occurs, people who have had the illness in the past will lose their immunity to the new strain, and vaccines against the original virus will also become less effective.

98. During routine infection prevention rounds in the Cardiac Catheterization Department, the IP notices that the air vents are dusty. The Nurse Manager is unsure of the cleaning schedule. The best action to take is: 1) Contact Environmental Services and request that the air vents be cleaned as soon as the room is available 2) Ask the nurse caring for the patient to dust the vents immediately 3) Reduce the number of air exchanges per hour until vent has been cleaned 4) Establish a monthly cleaning schedule a. 1, 2 b. 1, 4 c. 2, 3 d. 3, 4

98. B 1, 4 Rationale: Cardiac catheterization and associated diagnostic or therapeutic procedures bypass natural host defenses and thereby introduce the risk of infection. The air vent should be cleaned as soon as the room is empty to reduce the exposure of the patient to organisms. Air exchanges should provide three fresh air per 15 total air exchanges per hour. The air vents should be cleaned at least monthly.

8. A new Environmental Services employee has been asked to clean up a large blood spill on the floor in the OR. How should he proceed? a. He should mix an EPA-registered disinfectant with water in a bucket and mop up the spill b. He should place absorbent material over the spill and pour the correct dilution of disinfectant over the material for the recommended contact time c. He should pour undiluted bleach directly on the blood spill, wait 10 minutes, and then mop up the spill d. He should place absorbent material over the spill, dispose of the material after absorption, and then mop the floor with an EA-registered disinfectant

B He should place absorbent material over the spill and pour the correct dilution of disinfectant over the material for the recommended contact time Rationale: Cleaning of a large blood spill may be a risk for acquisition of bloodborne pathogens, so immediate inactivation of any pathogens before cleaning is important to reduce this risk. Absorbent material may be placed over the spill to contain it and the correct dilution of an EPA-registered disinfectant can be poured on the absorbent material to reduce the bioburden. After this, the absorbent materials can be gathered up and disposed of and the area can be cleaned.

9. Which of the following statements is true regarding an asymptomatic employee with a newly positive tuberculin skin test (TST) of 10 mm induration in a medium to high risk setting?1) The employee has latent tuberculosis (TB) infection 2) The employee is capable of transmitting TB to others 3) The employee is not infectious 4) The employee has TB disease a. 3, 4 b. 2, 4 c. 1, 3 d. 1, 2

C 1, 3 Rationale: Latent tuberculosis infection (LTBI) is the presence of M. tuberculosis organisms (tubercle bacilli) without symptoms or radiographic or bacteriologic evidence of TB. Approximately 90 to 95 percent of those infected are able to mount an immune response that halts the progression from LTBI to TB. Persons with LTBI are asymptomatic (they have no symptoms of TB) and are not infectious.

4. Using the surgical risk index to stratify the identified infections for the previous quarter, an IP would report which of the following case(s) as having a higher risk for developing a surgical site infection (SSI)? 1) An 80-year-old male with poor circulation who develops a donor site infection after a coronary artery bypass graft surgery that took 4 hours to perform 2) A 30-year-old female who has knee surgery to repair a torn anterior cruciate ligament (ACL) after a skiing accident 3) A 90-year-old female with insulin-dependent diabetes who has hip replacement surgery that takes 2.5 hours to per 4) A 27-year-old male with Crohn's disease who has colon resection that takes more than 4 hours to perform due to adhesions a. 1, 2 b. 2, 3 c. 3, 4 d. 1, 4

C 3, 4 Rationale: A surgical risk index is a score used to predict a surgical patient's risk of acquiring an SSI. The risk index score, ranging from 0 to 3, is the sum of the number of risk factors present among the following: • A patient with an ASA physical status classification score of 3, 4, or 5 • An operation classified as contaminated or dirty/infected • An operation lasting longer than the duration cut point in minutes, where the duration cut point varies by the type of operative procedure performed The higher the score by this index, the greater is the risk for subsequent SSI (see Table PE2-1).Patient 1 has a risk index of 1 and an ASA score of 3. Patient 2 has a risk index of 0. Patient 3 has an ASA score of 3 and an operation lasting longer than the duration cut point in minutes; her risk index would be 2. Patient 4 has a Class II procedure (contaminated) and an operation lasting longer than the duration cut point in minutes; his risk index is 2.

2. A classic sign of measles is: a. Kaposi sarcoma b. Stiff neck c. Koplik spots d. Bull's-eye rash

C Koplik spots Rationale: Measles is a highly communicable viral illness with prodromal fever, conjunctivitis, coryza, cough, and small spots with white or bluish-white centers on an erythematous base on the buccal mucosa. These small spots are called Koplik spots.

23. Infection prevention challenges related to body piercings and tattooing include all of the following except a. There have not been uniform licensure requirements for body art studios b. Reporting of infections to the Public Health Department is limited to bloodborne diseases c. The popularity of tattooing and body piercing has made it more socially acceptable d. Inspectors may not receive adequate training to conduct site inspections

C The popularity of tattooing and body piercing has made it more socially acceptable

5. A 47-year-old female bus driver is brought to the Emergency Department (ED) with a two-day history of fever, shortness of breath, and chest pain. She is diaphoretic and appears acutely ill. She is confused as to place and time. Temperature is 38°C (100.4°F), blood pressure is 88/60 mm Hg, pulse rate is 110/min, and respiration rate is 28/min. Coarse bronchial breath sounds are heard. She has had no recent known contact with ill persons. The leukocyte count is 15,000/μL (15 × 109/L). A chest radiograph shows a widened mediastinum and bilateral pleural effusions. Gram stain of a peripheral blood smear shows box car-shaped Gram-positive bacilli. A bioterrorism agent is suspected. Which of the following agents is most likely? a. Typhus fever b. Smallpox c. Tularemia d. Anthrax

D Anthrax Rationale: A biological attack, or bioterrorism, is the intentional release of viruses, bacteria, or other germs that can sicken or kill people, livestock, or crops. Bacillus anthracis, the bacteria that causes anthrax, is one of the most likely agents to be used in a biological attack because:• Anthrax spores are easily found in nature, can be produced in a lab, and can last for a long time in the environment• Anthrax makes a good weapon because it can be released quietly and without anyone knowing. The microscopic spores could be put into powders, sprays, food, and water. Because they are so small, individuals may not be able to see, smell, or taste them.• Anthrax has been used as a weapon before.There are three types of anthrax: cutaneous, gastrointestinal, and inhalational. Symptoms of inhalation anthrax include:• Fever and chills• Chest discomfort• Shortness of breath• Confusion or dizziness• Cough• Nausea, vomiting, or stomach pains• Headache• Sweats (often drenching)• Extreme tiredness• Body aches If inhalation anthrax is suspected, chest X-rays or computed tomography scans can confirm if the patient has mediastinal widening or pleural effusion, which are X-ray findings typically seen in patients with inhalation anthrax. The only way to confirm a diagnosis of anthrax is to either test directly for B. anthracis in a sample (blood, skin lesion swab, spinal fluid, or respiratory secretions) or measure antibodies or toxin in blood. Samples must be taken before the patient begins taking antibiotics.


Set pelajaran terkait

Writing Numbers in Scientific Notation & Standard Form, Some Multiplication and Division of Numbers in Scientific Notation

View Set

Finance 8 Personal Finance - Ch 5 Consumer Credit: Advantages, Disadvantages, Sources, and Costs

View Set

Sudden infant death syndrome (SIDS) AKA cot death

View Set